NUR 347 Final Q's

Pataasin ang iyong marka sa homework at exams ngayon gamit ang Quizwiz!

A 33-yr-old patient noticed a painless lump and heaviness in his scrotum during testicular self-examination. The nurse should provide the patient information on which diagnostic test? 1.Ultrasound 2.Cremasteric reflex 3.Doppler ultrasound 4.Transillumination with a flashlight

1

A 50-yr-old African American woman has a body mass index (BMI) of 35 kg/m2, type 2 diabetes mellitus, hypercholesterolemia, and irritable bowel syndrome (IBS). She is seeking assistance in losing weight because, "I have trouble stopping eating when I should, but I do not want to have bariatric surgery." Which drug therapy should the nurse question if it is prescribed for this patient? 1.Orlistat (Xenical) 2.Lorcaserin (Belviq) 3.Phentermine (Adipex-P) 4.Phentermine and topiramate (Qsymia)

1

A 71-yr-old patient with a diagnosis of benign prostatic hyperplasia (BPH) has been scheduled for a contact laser technique. What is the primary goal of this intervention? 1.Resumption of normal urinary drainage 2.Maintenance of normal sexual functioning 3.Prevention of acute or chronic renal failure 4.Prevention of fluid and electrolyte imbalances

1

A college student reports eye pain after studying for finals. What assessment should the nurse make first in determining the possible etiology of this eye pain? 1.Do you wear contacts? 2.Do you have any allergies? 3.Do you have double vision? 4.Describe the change in your vision.

1

A patient complaining of nausea receives a dose of metoclopramide. Which potential adverse effect should the nurse tell the patient to report? 1.Tremors 2.Constipation 3.Double vision 4.Numbness in fingers and toes

1

A patient had a gastric resection for stomach cancer. The nurse plans to teach the patient about decreased secretion of which hormone? 1.Gastrin 2.Secretin 3.Cholecystokinin 4.Gastric inhibitory peptide

1

A patient is prescribed intravenous (IV) gentamicin after repair of an intestinal perforation. The nurse should assess for which adverse effect of this medication? 1.Hearing loss 2.Exophthalmos 3.Conjunctivitis 4.Recurrent fever

1

A patient is scheduled for surgery with general anesthesia in 1 hour and is observed with a moist but empty water glass in his hand. Which assessment finding may indicate that the patient drank a glass of water? 1.Easily heard, loud gurgling in abdomen 2.High-pitched, hollow sounds in abdomen 3.Tenderness in left upper quadrant upon palpation 4.Flat abdomen without movement upon inspection

1

A patient reporting frequent vertigo is scheduled for electronystagmography to test vestibular function. Which instructions should the nurse provide to the patient before the procedure? 1.Eat a light meal before the procedure. 2.Avoid carbonated beverages before the procedure. 3.Take nothing by mouth for 3 hours before the procedure. 4.No special dietary restrictions are needed until after the procedure.

1

A patient with a family history of adenomatous polyposis had a colonoscopy with removal of multiple polyps. Which signs and symptoms should the nurse teach the patient to report immediately? 1.Fever and abdominal pain 2.Flatulence and liquid stool 3.Loudly audible bowel sounds 4.Sleepiness and abdominal cramps

1

A patient with a history of peptic ulcer disease has presented to the emergency department with severe abdominal pain and a rigid, boardlike abdomen. The health care provider suspects a perforated ulcer. Which interventions should the nurse anticipate? 1.Providing IV fluids and inserting a nasogastric (NG) tube 2.Administering oral bicarbonate and testing the patient's gastric pH level 3.Performing a fecal occult blood test and administering IV calcium gluconate 4.Starting parenteral nutrition and placing the patient in a high-Fowler's position

1

A patient with oral cancer is not eating. A small-bore feeding tube was inserted and the patient started on enteral feedings. Which patient goal would indicate improvement? 1.Weight gain of 1 kg in 1 week 2.Administer tube feeding at 25 mL/hr. 3.Consume 50% of clear liquid tray this shift. 4.Monitor for tube for placement and gastrointestinal residual.

1

After a vasectomy, what instruction should be included in discharge teaching? 1."Some secondary sexual characteristics may be lost after the surgery." 2."Use an alternative form of contraception until your semen is sperm free." 3."Erectile dysfunction may be present for several months after this surgery." 4."You will be uncomfortable, but you may safely have sexual intercourse today."

1

An older adult patient reports difficulty swallowing. Which age-related change does the nurse teach the patient about? 1.Xerostomia 2.Esophageal cancer 3.Decreased taste buds 4.Thinner abdominal wall

1

An older adult patient states they don't seem to hear well and have to ask people to repeat themselves. What should the nurse do first to determine the cause of the hearing loss? 1.Look for cerumen in the ear. 2.Assess for increased hair growth in the ear. 3.Tell the patient it is probably related to aging. 4.Ask the patient if he has fallen because of dizziness.

1

The nurse cares for a 34-yr-old woman after bariatric surgery. The nurse determines that discharge teaching related to diet is successful if the patient makes which statement? 1."A high-protein diet that is low in carbohydrates and fat will prevent diarrhea." 2."Food should be high in fiber to prevent constipation from the pain medication." 3."Three meals a day with no snacks between meals will provide optimal nutrition." 4."Fluid intake should be at least 2000 mL per day with meals to avoid dehydration."

1

The nurse instructs an obese 22-yr-old man with a sedentary job about the health benefits of an exercise program. The nurse evaluates that teaching is effective when the patient makes which statement? 1."The goal is to walk at least 10,000 steps every day of the week." 2."Weekend aerobics for 2 hours is better than exercising every day." 3."Aerobic exercise will increase my appetite and result in weight gain." 4."Exercise causes weight loss by decreasing my resting metabolic rate."

1

The nurse is caring for a 62-yr-old man after a transurethral resection of the prostate (TURP). Which instructions should the nurse include in the teaching plan? 1.Avoid straining during defecation. 2.Restrict fluids to prevent incontinence. 3.Sexual functioning will not be affected. 4.Prostate examinations are not needed after surgery.

1

The nurse is caring for a patient who complains of abdominal pain and hematemesis. Which new assessment finding(s) would indicate the patient is experiencing a decline in condition? 1.Pallor and diaphoresis 2.Ecchymotic peripheral IV site 3.Guaiac-positive diarrhea stools 4.Heart rate 90, respiratory rate 20, BP 110/60

1

The nurse is caring for a patient who is 5'6" tall and weighs 186 lb. The nurse has discussed reasonable weight loss goals and a low-calorie diet with the patient. Which statement made by the patient indicates a need for further teaching? 1."I will limit intake to 500 calories a day." 2."I will try to eat very slowly during mealtimes." 3."I'll try to pick foods from all of the basic food groups." 4."It's important for me to begin a regular exercise program."

1

The nurse is preparing a patient for a capsule endoscopy. What should the nurse ensure is included in the preparation? 1.Ensure the patient understands the required bowel preparation. 2.Have the patient return to the procedure room for removal of the capsule. 3.Teach the patient to maintain a clear liquid diet throughout the procedure. 4.Explain to the patient that conscious sedation will be used during capsule placement.

1

The nurse is providing discharge instructions for a patient using contact lenses who is diagnosed with bacterial conjunctivitis. What is most important for the nurse to include in the instructions? 1.Discard all opened or used lens care products. 2.Disinfect contact lenses by soaking in a cleaning solution for 48 hours. 3.Put all used cosmetics in a plastic bag for 1 week to kill any bacteria before reusing. 4.Disinfect all lens care products with the prescribed antibiotic drops for 1 week after infection.

1

The nurse teaches a 50-yr-old woman who has a body mass index (BMI) of 39 kg/m2 about weight loss. Which dietary change would be most appropriate to recommend? 1.Decrease fat intake and control portion size. 2.Increase vegetables and decrease fluid intake. 3.Increase protein intake and avoid carbohydrates. 4.Decrease complex carbohydrates and limit fiber.

1

The nurse teaches senior citizens at a community center how to prevent food poisoning at social events. Which community member statement reflects accurate understanding? 1."Pasteurized juices and milk are safe to drink." 2."Alfalfa sprouts are safe if rinsed before eating." 3."Fresh fruits do not need to be washed before eating." 4."Ground beef is safe to eat if cooked until it is brown."

1

The patient reports a loss of central vision. What test should the nurse teach the patient about to identify changes in macular function? 1.Amsler grid test 2.B-scan ultrasonography 3.Fluorescein angiography 4.Intraocular pressure testing with Tono-Pen

1

The results of a patient's recent endoscopy indicate the presence of peptic ulcer disease (PUD). Which teaching point should the nurse provide to the patient based on this new diagnosis? 1."It would be beneficial for you to eliminate drinking alcohol." 2."You'll need to drink at least two to three glasses of milk daily." 3."Many people find that a minced or pureed diet eases their symptoms of PUD." 4."Taking medication will allow you to keep your present diet while minimizing symptoms."

1

Which finding related to primary open-angle glaucoma would the nurse expect to find when reviewing a patient's history and physical examination report? 1.Absence of pain or pressure 2.Blurred vision in the morning 3.Seeing colored halos around lights 4.Eye pain accompanied with nausea and vomiting

1

A patient is scheduled to receive "Colace 100 mg PO." The patient asks to take the medication in liquid form, and the nurse obtains an order for the change. The available syrup contains 150 mg/15 mL. Calculate how many milliliters the nurse should administer.

10 mL

The patient has parenteral nutrition (PN) infusing with amino acids and dextrose. During shift change, the nurse reports the tubing, bag, and dressing were changed 20 hours ago. What care should the incoming nurse plan to deliver (select all that apply.)? Select all that apply. 1.Giving the patient insulin if needed 2.Ensuring that the next bag has been ordered 3.Checking amount of solution left in the bag 4.Assessing the insertion site and change the tubing 5.Verifying the accuracy of the new solution and ingredients

1235

The stable patient has a gastrostomy tube for enteral feeding. Which care could the RN delegate to the LPN (select all that apply.)? Select all that apply. 1.Administer bolus or continuous feedings. 2.Evaluate the nutritional status of the patient. 3.Administer medications through the gastrostomy tube. 4.Monitor for complications related to the tube and enteral feeding. 5.Teach the caregiver about feeding via the gastrostomy tube at home.

13

A malnourished patient has been diagnosed with protein deficiency. Which complications should the nurse anticipate (select all that apply.)? Select all that apply. 1.Edema 2.Asthma 3.Anemia 4.Malabsorption syndrome 5.Impaired wound healing 6.Gastrointestinal bleeding

135

A community health nurse is conducting an initial assessment of a new patient. Which assessments should the nurse include when screening the patient for metabolic syndrome (select all that apply.)? Select all that apply. 1.Blood pressure 2.Resting heart rate 3.Physical endurance 4.Waist circumference 5.Fasting blood glucose

145

A 73-yr-old male patient admitted for total knee replacement states during the health history interview that he has no problems with urinary elimination except that the "stream is less than it used to be." The nurse should give anticipatory guidance regarding what condition? 1.A tumor of the prostate 2.Benign prostatic hyperplasia 3.Bladder atony because of age 4.Age-related altered innervation of the bladder

2

A frail older adult with recent severe weight loss is instructed to eat a high-protein, high-calorie diet at home. Which foods would the nurse suggest for breakfast? 1.Orange juice and dry toast 2.Oatmeal with butter and cream 3.Waffles with fresh strawberries 4.Banana and unsweetened yogurt

2

A male patient complains of fever, dysuria, and cloudy urine. What additional information may indicate that these manifestations may be something other than a urinary tract infection (UTI)? 1.E. coli bacteria in his urine 2.A very tender prostate gland 3.Complaints of chills and rectal pain 4.Complaints of urgency and frequency

2

A patient complains of intermittent eye dryness. Which question should the nurse ask the patient to determine the etiology of this symptom? 1."Do you take ginkgo to treat asthma or tinnitus?" 2."What do you take if you have allergy symptoms?" 3."Are you taking propranolol for an anxiety disorder?" 4."How long have you been taking prednisone (Deltasone)?"

2

A patient has ptosis resulting from myasthenia gravis. Which assessment finding would the nurse expect to see in this patient? 1.Redness and swelling of the conjunctiva 2.Drooping of the upper lid margin in one or both eyes 3.Redness, swelling, and crusting along the eyelid margins 4.Small, superficial white nodules along the eyelid margin

2

A patient is admitted with anorexia nervosa and a serum potassium level of 2.4 mEq/L. What complication is most important for the nurse to observe for in this patient? 1.Muscle weakness 2.Cardiac dysrhythmias 3.Increased urine output 4.Anemia and leukopenia

2

A patient is being admitted with anorexia nervosa. Which clinical manifestations should the nurse anticipate? 1.Sensitivity to heat, fatigue, and polycythemia 2.Hair loss; dry, yellowish skin; and constipation 3.Tented skin turgor, hyperactive reflexes, and diarrhea 4.Dysmenorrhea, hypoactive bowel sounds, and hunger

2

A patient is diagnosed with severe myopia. Which type of correction is the patient planning to have if they state, "I can't wait to be able to see after they implant a contact lens over my lens"? 1.Photorefractive keratectomy (PRK) 2.Phakic intraocular lenses (phakic IOLs) 3.Refractive intraocular lens (refractive IOL) 4.Laser-assisted in situ keratomileusis (LASIK)

2

A patient is scheduled for a percutaneous transhepatic cholangiography to restore biliary drainage. The nurse discusses the patient's health history and is most concerned if the patient makes which statement? 1."I am allergic to bee stings." 2."My tongue swells when I eat shrimp." 3."I have had epigastric pain for 2 months." 4."I have a pacemaker because my heart rate was slow."

2

A patient is seeking emergency care after choking on a piece of steak. The nursing assessment reveals a history of alcoholism, cigarette smoking, and hemoptysis. Which diagnostic study is most likely to be performed on this patient? 1.Barium swallow 2.Endoscopic biopsy 3.Capsule endoscopy 4.Endoscopic ultrasonography

2

A patient was admitted with a fractured hip after being found on the floor of her home. She was extremely malnourished and started on parenteral nutrition (PN) 3 days ago. Which assessment finding would be of most concern to the nurse? 1.Blood glucose level of 125 mg/dL 2.Serum phosphate level of 1.9 mg/dL 3.White blood cell count of 10,500/μL 4.Serum potassium level of 4.6 mEq/L

2

A patient who cannot afford enough food for her family states she only eats after her children have eaten. At a clinic visit, she reports bleeding gums; loose teeth; and dry, itchy skin. Which vitamin deficiency would the nurse suspect? 1.Folic acid 2.Vitamin C 3.Vitamin D 4.Vitamin K

2

A patient who has sustained severe burns in a motor vehicle accident is starting parenteral nutrition (PN). Which principle should guide the nurse's administration of PN? 1.Administration of PN requires clean technique. 2.Central PN requires rapid dilution in a large volume of blood. 3.Peripheral PN delivery is preferred over the use of a central line. 4.Only water-soluble medications may be added to the PN by the nurse.

2

A patient with Ménière's disease had decompression of the endolymphatic sac to reduce the frequent and incapacitating attacks being experienced. What should the nurse include in the discharge teaching for this patient? 1.Airplane travel will be more comfortable now. 2.Avoid sudden head movements or position changes. 3.Cough or blow the nose to keep the Eustachian tubes clear. 4.Take antihistamines, antiemetics, and sedatives for recovery.

2

After administering a dose of promethazine to a patient with nausea and vomiting, what medication side effect does the nurse explain is common and expected? 1.Tinnitus 2.Drowsiness 3.Reduced hearing 4.Sensation of falling

2

An older adult patient is seen in the primary care provider's office for a well check complains of difficulty swallowing. What common effect of aging should the nurse assess for as a possible cause? 1.Anosmia 2.Xerostomia 3.Hypochlorhydria 4.Salivary gland tumor

2

Inspection of an older patient's mouth reveals the presence of white, curd-like lesions on the patient's tongue. What does the nurse recognize is the most likely etiology for this abnormal assessment finding? 1.Herpesvirus 2.Candida albicans 3.Vitamin deficiency 4.Irritation from ill-fitting dentures

2

The nurse has completed initial instruction with a patient regarding a weight loss program. The nurse determines that the teaching has been effective when the patient makes which statement? 1."I plan to lose 4 lb a week until I have lost the 60-lb goal." 2."I will keep a diary of weekly weights to illustrate my weight loss." 3."I will restrict my carbohydrate intake to less than 30 g/day to maximize weight loss." 4."I should not exercise more than my program requires because increased activity increases the appetite."

2

The nurse is assessing a patient's medical history. What aspects of the patient's medical history are most likely to have potential consequences for the patient's visual system? 1.Hypothyroidism and polycythemia 2.Hypertension and diabetes mellitus 3.Atrial fibrillation and atherosclerosis 4.Vascular dementia and chronic fatigue

2

The nurse is caring for a 45-yr-old woman with a herniated lumbar disc. The patient realizes that weight loss is necessary to lessen back strain. The patient is 5'6" tall and weighs 186 lb (84.5 kg) with a body mass index (BMI) of 28 kg/m2. The nurse explains this measurement places her in which weight category? 1.Obese 2.Overweight 3.Severely obese 4.Normal weight

2

The nurse is caring for a patient treated with IV fluid therapy for severe vomiting. As the patient recovers and begins to tolerate oral intake, which food choice would be most appropriate? 1.Iced tea 2.Dry toast 3.Hot coffee 4.Plain yogurt

2

The nurse is caring for a postoperative patient who has just vomited yellow green liquid and reports nausea. Which action would be an appropriate nursing intervention? 1.Offer the patient an herbal supplement such as ginseng. 2.Apply a cool washcloth to the forehead and provide mouth care. 3.Take the patient for a walk in the hallway to promote peristalsis. 4.Discontinue any medications that may cause nausea or vomiting.

2

The nurse is evaluating the nutritional status of a patient undergoing radiation treatment for oropharyngeal cancer. Which laboratory test would best indicate the patient has protein-calorie malnutrition (PCM)? 1.Serum transferrin 2.Serum prealbumin 3.C-reactive protein (CRP) 4.Alanine transaminase (ALT)

2

The nurse is examining a patient's ear in the clinic to determine if recent treatment for acute otitis media has been effective. Which assessment finding indicates resolution of the middle ear infection? 1.Fenestrations are visible in the tympanic membrane. 2.Tympanic membrane is gray, shiny, and translucent. 3.Cone of light is not visible on the tympanic membrane. 4.Tympanic membrane is blue and bulging with no landmarks.

2

The nurse is providing care for a patient with loss of hearing acuity over the past several years. Which statement by the nurse is most accurate? 1."This is often due to an infection that will resolve on its own." 2."Many people experience an age-related decline in their hearing." 3."This is likely an effect of your medications. Try stopping them for a few days." 4."You can likely accommodate for your hearing loss with a few small changes in your routine."

2

The nurse is teaching a patient with type 1 diabetes mellitus who had surgery to revise a lower leg stump with a skin graft about nutrition. What food should the nurse teach the patient to eat to best facilitate healing? 1.Nonfat milk 2.Chicken breast 3.Fortified oatmeal 4.Olive oil and nuts

2

The patient informs the nurse that he has a "sty" that has been present for some time on the upper eyelid and reports using warm moist compresses with no improvement. What is the best response by the nurse? 1."Go to the pharmacy to get some eye drops." 2."Come in so the ophthalmologist can assess the lesion." 3."The health care provider will need to inject it with an antibiotic." 4."Wash the eyelid margins with baby shampoo to remove the crusting."

2

The patient receiving chemotherapy rings the call bell and reports the onset of nausea. The nurse should prepare an as-needed dose of which medication? 1.Zolpidem 2.Ondansetron 3.Dexamethasone 4.Morphine sulfate

2

The patient who is admitted with a diagnosis of diverticulitis and a history of irritable bowel disease and gastroesophageal reflux disease (GERD) has received a dose of Mylanta 30 mL PO. The nurse will determine the medication was effective when which symptom has been resolved? 1.Diarrhea 2.Heartburn 3.Constipation 4.Lower abdominal pain

2

The triage nurse at an ambulatory clinic receives a call from an individual with possible metal fragments in both eyes. Which instructions would the nurse provide for emergency care of this possible eye injury? 1."Remove any visible metal fragments." 2."Apply a loose dressing over your eyes." 3."Rinse your eyes immediately with water." 4."Keep your eyes open to allow tears to form."

2

When examining the patient's ear with an otoscope, the nurse observes discharge in the canal and the patient reports pain with the examination. What should the nurse next assess the patient for? 1.Sebaceous cyst 2.Swimmer's ear 3.Metabolic disorder 4.Serous otitis media

2

When teaching a patient about the pathophysiology related to open-angle glaucoma, which statement is most appropriate? 1."The retinal nerve is damaged by an abnormal increase in the production of aqueous humor." 2."There is decreased draining of aqueous humor in the eye, causing pressure damage to the optic nerve." 3."The lens enlarges with normal aging, pushing the iris forward, which then covers the outflow channels of the eye." 4."There is a decreased flow of aqueous humor into the anterior chamber by the lens of the eye blocking the papillary opening."

2

When using the otoscope, the nurse is unable to see the landmarks or light reflex of the tympanic membrane. The tympanic membrane is bulging and red. What does the nurse determine is most likely occurring in the patient's ear? 1.Swimmer's ear 2.Acute otitis media 3.Impacted cerumen 4.Chronic otitis media

2

Which focused assessments would have priority in the care of a patient recently started on parenteral nutrition (PN)? 1.Skin integrity and skin turgor 2.Electrolyte levels and daily weights 3.Auscultation of lung and bowel sounds 4.Peripheral edema and level of consciousness

2

An acoustic neuroma is removed from a patient. The nurse instructs the patient about tumor recurrence. What should the nurse instruct the patient to monitor (select all that apply.)? Select all that apply. 1.Lack of coordination 2.Episodes of dizziness 3.Worsening of hearing 4.Inability to close the eye 5.Clear drainage from the nose

2,3,4

The nurse should recognize that the liver performs which functions (select all that apply.)? Select all that apply. 1.Bile storage 2.Detoxification 3.Protein metabolism 4.Steroid metabolism 5.Red blood cell (RBC) destruction

2,3,4,5

The nurse is caring for a patient after bariatric surgery. What should be included in the plan of care (select all that apply.)? Select all that apply. 1.Teach the patient to increase carbohydrate intake. 2.Assess for incisional pain versus anastomosis leak. 3.Maintain elevation of the head of bed at 35-45 degrees. 4.Monitor for vomiting that is a common complication. 5.Instruct the patient to consume liquids frequently during meals. 6.Assist with early independent ambulation during hospitalization.

2346

A 74-yr-old female patient with osteoporosis is diagnosed with gastroesophageal reflux disease (GERD). Which over-the-counter medication to treat GERD should be used with caution? 1.Sucralfate 2.Cimetidine 3.Omeprazole 4.Metoclopramide

3

A hospitalized older patient reports his foreskin is retracted and will not return to normal. Which action is the priority? 1.Start antibiotics. 2.Apply ice to reduce swelling. 3.Attempt to move the foreskin over the glans. 4.Call the physician to prepare for circumcision.

3

A patient has a sliding hiatal hernia. What nursing intervention will reduce the symptoms of heartburn and dyspepsia? 1.Keeping the patient NPO 2.Putting the bed in the Trendelenburg position 3.Having the patient eat 4 to 6 smaller meals each day 4.Giving various antacids to determine which one works for the patient

3

A patient is one day postoperative after a transurethral resection of the prostate (TURP). Which event is an unexpected finding? 1.Requires two tablets of Tylenol #3 during the night 2.Complains of fatigue and claims to have minimal appetite 3.Continuous bladder irrigation (CBI) infusing, but output has decreased 4.Expressed anxiety about his planned discharge home the following day

3

A patient is recovering from a motor vehicle crash that resulted in blindness. The patient is withdrawn and refuses to get out of bed. What is the nurse's priority goal for this patient? 1.Use suitable coping strategies to reduce stress. 2.Identify patient's strengths and support system. 3.Verbalize feelings related to visual impairment. 4.Transition successfully to the sudden vision loss.

3

A patient is suspected of having acute pancreatitis after presenting to the emergency department with severe abdominal pain. Which laboratory result would best indicate the presence of acute pancreatitis? 1.Gastric pH of 1.4 2.Blood glucose of 104 3.Serum amylase of 420 U/L 4.Serum potassium of 3.5 mEq/L

3

A patient newly diagnosed with glaucoma asks the nurse what has made the pressure in the eyes so high. Which is the nurse's most accurate response? 1.Back pressure from cardiac congestion causes corneal edema. 2.Cerebral venous dilation prevents normal interstitial fluid resorption. 3.Increased production of aqueous humor or blocked drainage increases pressure. 4.Congenital anomalies of the lacrimal gland or duct obstruct the passage of tears.

3

A patient who had a gastroduodenostomy (Billroth I operation) for stomach cancer reports generalized weakness, sweating, palpitations, and dizziness 15 to 30 minutes after eating. What long-term complication does the nurse suspect is occurring? 1.Malnutrition 2.Bile reflux gastritis 3.Dumping syndrome 4.Postprandial hypoglycemia

3

A patient who has dysphagia after a stroke is receiving enteral feedings through a percutaneous endoscopic gastrostomy (PEG). What intervention should the nurse integrate into the plan of care? 1.Use 30 mL of normal saline to flush the tube every 4 hours. 2.Avoid flushing the tube any time the patient is receiving continuous feedings. 3.Flush the tube before and after feedings if the patient's feedings are intermittent. 4.Flush the PEG with 100 mL of sterile water before and after medication administration.

3

A patient with abdominal pain is being prepared for surgery to make an incision into the common bile duct to remove stones. What procedure will the nurse prepare the patient for? 1.Colectomy 2.Cholecystectomy 3.Choledocholithotomy 4.Choledochojejunostomy

3

A patient working in a noisy factory reports being off balance when standing or walking but not while lying down. What term will the nurse use to document this patient's symptoms? 1.Vertigo 2.Syncope 3.Dizziness 4.Nystagmus

3

After administration of a dose of metoclopramide, which patient assessment finding would show the medication was effective? 1.Decreased blood pressure 2.Absence of muscle tremors 3.Relief of nausea and vomiting 4.No further episodes of diarrhea

3

During a health history, a 43-yr-old teacher complains of increasing difficulty reading printed materials for the past year. What change related to aging does the nurse suspect? 1.Myopia 2.Hyperopia 3.Presbyopia 4.Astigmatism

3

In developing a weight reduction program with a 45-yr-old female patient who weighs 197 lb, the nurse encourages the patient to set a weight loss goal of how many pounds in 4 weeks? 1.1 to 2 2.3 to 5 3.4 to 8 4.5 to 10

3

In developing an effective weight reduction plan for an overweight patient who expresses willingness to try to lose weight, which factor should the nurse assess first? 1.The length of time the patient has been obese 2.The patient's current level of physical activity 3.The patient's social, emotional, and behavioral influences on obesity 4.Anthropometric measurements, such as body mass index and skinfold thickness

3

Otoscopic examination of the patient's left ear indicates the presence of an exostosis. What does the nurse prepare to teach the patient about regarding the growth? 1.Surgery 2.Electrocochleography 3.Monitoring of the growth 4Irrigation of the ear canal

3

The nurse determines a patient has experienced the beneficial effects of therapy with famotidine when which symptom is relieved? 1.Nausea 2.Belching 3.Epigastric pain 4.Difficulty swallowing

3

The nurse is assessing a patient admitted with a possible bowel obstruction. Which assessment finding would be expected in this patient? 1.Tympany to abdominal percussion 2.Aortic pulsation visible in epigastric region 3.High-pitched sounds on abdominal auscultation 4.Liver border palpable 1 cm below the right costal margin

3

The nurse is performing an abdominal assessment for a patient. Which assessment technique by the nurse is most accurate? 1.Palpate the abdomen before auscultation. 2.Percuss the abdomen before auscultation. 3.Auscultate the abdomen before palpation. 4.Perform deep palpation before light palpation.

3

The nurse is preparing to administer timolol eye drops for treatment of glaucoma. What statement made by the patient would cause the nurse to hold the medication and report to the health care provider? 1."I have sinusitis." 2."I have migraine headaches a lot." 3."I have chronic obstructive pulmonary disease." 4."I have a history of chronic urinary tract infections."

3

The nurse is providing care for a patient who is a strict vegetarian. Which dietary choices would the nurse recommend to prevent iron deficiency? 1.Brown rice and kidney beans 2.Cauliflower and egg substitutes 3.Soybeans and hot breakfast cereal 4.Whole-grain bread and citrus fruits

3

The nurse is providing discharge teaching to a patient with type 2 diabetes after a scleral buckling procedure. Which statement, if made by the patient, indicates that the discharge teaching is effective? 1."I doubt my other eye will ever be affected." 2."I can expect severe pain after this procedure." 3."I should avoid lifting heavy objects and straining." 4."The procedure will correct my vision immediately."

3

The nurse recognizes that the majority of patients' caloric needs should come from which source? 1.Fats 2.Proteins 3.Polysaccharides 4.Monosaccharides

3

The nurse teaches a 30-yr-old man with a family history of prostate cancer about dietary factors associated with prostate cancer. The nurse determines that teaching is successful if the patient selects which menu? 1.Grilled steak, French fries, and vanilla shake 2.Hamburger with cheese, pudding, and coffee 3.Baked chicken, peas, apple slices, and skim milk 4.Grilled cheese sandwich, onion rings, and hot tea

3

The patient has a history of cardiovascular disease and has developed erectile dysfunction. He is frustrated because he is taking nitrates and cannot take erectogenic medications. What should the nurse do first? 1.Give the patient choices for penile implant surgery. 2.Recommend counseling for the patient and his partner. 3.Obtain a thorough sexual, health, and psychosocial history. 4.Assess levels of testosterone, prolactin, luteinizing hormone, and thyroid hormones.

3

The patient is having an esophagoenterostomy with anastomosis of a segment of the colon to replace the resected portion. What initial postoperative care should the nurse expect when this patient returns to the nursing unit? 1.Turn, deep breathe, cough, and use spirometer every 4 hours. 2.Maintain an upright position for at least 2 hours after eating. 3.NG will have bloody drainage and it should not be repositioned. 4.Keep in a supine position to prevent movement of the anastomosis.

3

The patient with chronic gastritis is being put on a combination of medications to eradicate Helicobacter pylori. Which drugs does the nurse know will probably be used? 1.Antibiotic(s), antacid, and corticosteroid 2.Antibiotic(s), aspirin, and antiulcer/protectant 3.Antibiotic(s), proton pump inhibitor, and bismuth 4.Antibiotic(s) and nonsteroidal antiinflammatory drugs (NSAIDs)

3

To monitor the progression of decreased urinary stream, the nurse should encourage which type of regular screening? 1.Uroflowmetry 2.Transrectal ultrasound 3.Digital rectal examination (DRE) 4.Prostate-specific antigen (PSA) monitoring

3

When administering eye drops to a patient with glaucoma, which nursingmeasure is most appropriate to minimize systemic effects of the medication? 1.Apply pressure to each eyeball for a few seconds after administration. 2.Have the patient close the eyes and move them back and forth several times. 3.Have the patient put pressure on the inner canthus of the eye after administration. 4.`Have the patient try to blink out excess medication immediately after administration.

3

When assessing an adult patient's external ear canal and tympanum, what assessment techniques should the nurse use? 1.Ask the patient to tip his or her head toward the nurse. 2.Identify a pearl gray tympanic membrane as a sign of infection. 3.Gently pull the auricle up and backward to straighten the canal. 4.Identify a normal light reflex by the appearance of irregular edges.

3

When caring for the patient with heart failure, the nurse knows that which gastrointestinal process is most dependent on cardiac output and may affect the patient's nutritional status? 1.Ingestion 2.Digestion 3.Absorption 4.Elimination

3

When planning care for a patient with disturbed sensory perception related to increased intraocular pressure caused by primary open-angle glaucoma, what nursing action would be a priority? 1.Recognizing that eye damage caused by glaucoma can be reversed in the early stages 2.Giving anticipatory guidance about the eventual loss of central vision that will occur 3.Encouraging compliance with drug therapy for the glaucoma to prevent loss of vision 4.Managing the pain experienced by patients with glaucoma that persists until the optic nerve atrophies

3

Which patient would be at highest risk for developing oral candidiasis? 1.A 74-yr-old patient who has vitamin B and C deficiencies 2.A 22-yr-old patient who smokes 2 packs of cigarettes per day 3.A 32-yr-old patient who is receiving ciprofloxacin for 3 weeks 4.A 58-yr-old patient who is receiving amphotericin B for 2 days

3

Which task can the nurse delegate to an unlicensed assistive personnel (UAP) in the care of a patient who has recently undergone prostatectomy? 1.Assessing the patient's incision 2.Irrigating the patient's urinary catheter 3.Reporting complaints of pain or bladder spasms 4.Evaluating the patient's pain and selecting analgesia

3

The patient has a low-grade carcinoma on the left lateral aspect of the prostate gland and has been on "watchful waiting" status for 5 years. Six months ago, his last prostate-specific antigen (PSA) level was 5 ng/mL. Which manifestations indicate prostate cancer may be extending and require a change in the plan of care (select all that apply.)? Select all that apply. 1.Casts in his urine 2.Presence of α-fetoprotein 3.Serum PSA level 10 ng/mL 4.Onset of erectile dysfunction 5.Nodularity of the prostate gland 6.Development of a urinary tract infection

3,5

A patient is scheduled for a corneal transplant and is concerned regarding the difficulty with vision that may last for up to 12 months after the transplant. What is the best response by the nurse? 1.If the transplant is done soon after the donor dies, there will not be as much trouble recovering vision. 2.The astigmatism the patient is experiencing may be corrected with glasses or rigid contact lenses. 3.Increasing the amount of light and using a magnifier to read will be helpful if a transplant is not wanted. 4.There are newer procedures in which only the damaged cornea epithelial layer is replaced, and they have a faster recovery.

4

A patient received a small-bore nasogastric (NG) tube after a laryngectomy. Which action has the highest priority before initiating enteral feedings? 1.Testing aspirated fluid pH 2.Auscultating while instilling air 3.Elevating head of bed to 40 degrees 4.Verifying NG tube placement on x-ray

4

A patient reports severe pain when the nurse assesses for rebound tenderness. What may this assessment finding indicate? 1.Hepatic cirrhosis 2.Hypersplenomegaly 3.Gallbladder distention 4.Peritoneal inflammation

4

A patient was admitted with epigastric pain because of a gastric ulcer. Which patient assessment warrants an urgent change in the nursing plan of care? 1.Back pain 3 or 4 hours after eating a meal 2.Chest pain relieved with eating or drinking water 3.Burning epigastric pain 90 minutes after breakfast 4.Rigid abdomen and vomiting following indigestion

4

A patient was involved in a motor vehicle crash and reports an inability to have a bowel movement. What is the best response by the nurse? 1."You are just too nervous to eat or drink, so there is no stool." 2."Your parasympathetic nervous system is now working to slow the GI tract." 3."The circulation in the GI system has been increased, so less waste is removed." 4."Your sympathetic nervous system was activated, so there is slowing of the GI tract".

4

A patient who is unable to swallow because of progressive amyotrophic lateral sclerosis is prescribed enteral nutrition through a newly placed gastrostomy tube. Which task is appropriate for the nurse to delegate to unlicensed assistive personnel (UAP)? 1.Irrigate the tube between feedings. 2.Provide wound care at the gastrostomy site. 3.Administer prescribed liquid medications through the tube. 4.Position the patient with a 45-degree head of bed elevation.

4

A patient with poor visual acuity is diagnosed with age-related macular degeneration (AMD). Which nursing action should be the nurse's priority? 1.Teach about visual enhancement techniques. 2.Teach nutritional strategies to improve vision. 3.Assess coping strategies and support systems. 4.Assess impact of vision on normal functioning.

4

A patient with septic shock is receiving multiple medications. Which intravenous (IV) medication is most likely to cause a hearing loss? 1.Dopamine 2.Ampicillin 3.Aspirin 4.Vancomycin

4

After identifying that a patient has possible nutritional deficits, which action will the nurse perform next? 1.Provide supplements between meals. 2.Encourage eating meals with others. 3.Have family bring in food from home. 4.Complete a full nutritional assessment.

4

At the first visit to the clinic, the female patient with a BMI of 29 kg/m2 tells the nurse that she does not want to become obese. Which question used for assessing weight issues would be most effective? 1."What factors contributed to your current body weight?" 2."How is your overall health affected by your body weight?" 3."What is your history of gaining weight and losing weight?" 4."In what ways are you interested in managing your weight differently?"

4

During the course of an interview to assess vision, a patient complains of dry eyes. What should the nurse implement next? 1.Assess for contact lenses. 2.Suggest saline eye drops. 3.Ask about eyeglass usage. 4.Check the medication list.

4

In the immediate postoperative period a nurse cares for a severely obese 72-yr-old man who had surgery for repair of a lower leg fracture. Which assessment is most important? 1.Cardiac rhythm 2.Surgical dressing 3.Postoperative pain 4.Oxygen saturation

4

The nurse instructs a patient prescribed dipivefrin eye drops to manage chronic open-angle glaucoma. Which statement, if made by the patient to the nurse, indicates that further teaching is needed? 1."The eye drops could cause a fast heart rate and high blood pressure." 2."I will need to take the eye drops twice a day for at least 2 to 3 months." 3."I may experience eye discomfort and redness from the use of these eye drops." 4."I will apply gentle pressure on the inside corner of my eye after each eye drop."

4

The nurse is assessing an older adult patient who has just been transferred to the long-term care facility. Which assessment question will best allow the nurse to assess for the presence of presbycusis? 1."Do you ever experience any ringing in your ears?" 2."Have you ever fallen down because you became dizzy?" 3."Do you ever have pain in your ears when you're chewing or swallowing?" 4."Have you noticed any change in your hearing in recent months and years?"

4

The nurse is performing a focused abdominal assessment of a patient who has been recently admitted. In order to palpate the patient's liver, where should the nurse palpate the patient's abdomen? 1.Left lower quadrant 2.Left upper quadrant 3.Right lower quadrant 4.Right upper quadrant

4

The nurse is reviewing the home medication list for a patient admitted with suspected hepatic failure. Which medication reviewed by the nurse could cause hepatotoxicity? 1.Digoxin 2.Nitroglycerin 3.Ciprofloxacin 4.Acetaminophen

4

The nurse is reviewing the laboratory test results for a patient with metastatic lung cancer who was admitted with a diagnosis of malnutrition. The serum albumin level is 4.0 g/dL, and prealbumin is 10 mg/dL. How will the nurse interpret these results? 1.The albumin level is normal therefore the patient does not have protein malnutrition. 2.The albumin level is increased, which is common in patients with cancer who have malnutrition. 3.Both the serum albumin and prealbumin levels are reduced, consistent with the diagnosis of malnutrition. 4.Although the serum albumin level is normal, the prealbumin level more accurately reflects the patient's nutritional status.

4

The nurse is teaching a group of college students how to prevent food poisoning. Which comment shows an understanding of foodborne illness protection? 1."Eating raw cookie dough from the package is a great snack when you do not have time to bake." 2."Since we only have one cutting board, we can cut up chicken and salad vegetables at the same time." 3."To save refrigerator space, leftover food can be kept on the counter if it is in sealed containers." 4."When the cafeteria gave me a pink hamburger, I sent it back and asked for a new bun and clean plate."

4

The nurse is teaching a patient about timolol eye drops for the treatment of glaucoma. What statement made by the patient demonstrates that teaching was effective? 1."I may feel some palpitations after instilling these eye drops." 2."I should withhold this medication if my blood pressure becomes elevated." 3."I should keep my eyes closed for 15 minutes after instilling these eye drops." 4."I may have some temporary blurring of vision after instilling these eye drops."

4

The nurse is teaching a patient with glaucoma about administration of pilocarpine. What statement is important for the nurse to include during the instructions? 1."Prolonged eye irritation is an expected adverse effect of this medication." 2."This medication will help to raise intraocular pressure to a near normal level." 3."This medication needs to be continued for at least 5 years after your initial diagnosis." 4."It is important not to do activities requiring visual acuity immediately after administration."

4

The severely obese patient has elected to have the Roux-en-Y gastric bypass (RYGB) procedure. The nurse will know the patient understands the preoperative teaching when the patient makes which statement? 1."This surgery will preserve the function of my stomach." 2."This surgery will remove the fat cells from my abdomen." 3."This surgery can be modified whenever I need it to be changed." 4."This surgery decreases how much I can eat and how many calories I can absorb."

4

When administering a scheduled dose of pilocarpine, in which area should the nurse place the drops? 1.Inner canthus 2.Outer canthus 3.Center of the eyeball 4.Lower conjunctival sac

4

Which patient has the morbidity risk? 1.Male 6 ft, 1 in. tall; BMI 29 kg/m2 2.Female 5 ft, 6 in. tall; weight 150 lb 3.Male with waist circumference 46 in 4.Female 5 ft, 10 in. tall; obesity class III

4

Which patient is at risk for developing metabolic syndrome? 1.A 62-yr-old white man who has coronary artery disease with chronic stable angina 2.A 54-yr-old Hispanic woman who is sedentary and has nephrogenic diabetes insipidus 3.A 27-yr-old Asian American woman who has preeclampsia and gestational diabetes mellitus 4.A 38-yr-old Native American man who has diabetes mellitus and elevated hemoglobin A1C

4

If the patient in shock is to receive 1000 mL of normal saline in 2 hours, at what rate should the infusion pump be set?__ mL per hour

500 mL/hr

The nurse is caring for a 26-yr-old patient who is being discharged after an induced abortion. Which statement should the nurse include in discharge teaching? A. "Avoid sexual intercourse for 2 weeks." B. "Heavy bleeding is expected for 24 hours." C. "A temperature of 101oF (38.9oC) is normal" D. "Birth control pills should not be taken for 30 days."

A "Avoid sexual intercourse for 2 weeks." After an abortion, teach the patient to avoid intercourse for 2 weeks. Contraception can be started the day of the procedure. Symptoms of possible complications include a fever and abnormal vaginal bleeding. These symptoms should be reported immediately.

When evaluating the patient's understanding about the care of the ileostomy, which statement by the patient indicates the patient needs more teaching? a. "I will be able to regulate when I have stools." b. "I will be able to wear the pouch until it leaks." c. "The drainage from my stoma can damage my skin." d. "Dried fruit and popcorn must be chewed very well."

A "I will be able to regulate when I have stools." An ileostomy is in the ileum and drains liquid stool frequently, unlike a colostomy, which has more formed stool the farther distal the ostomy is in the colon. The ileostomy pouch is usually worn for 4 to 7 days or until it leaks. It must be changed immediately if it leaks because the drainage is very irritating to the skin. To avoid obstruction, popcorn, dried fruit, coconut, mushrooms, olives, stringy vegetables, food with skin, and meats with casings must be chewed extremely well before swallowing because of the narrow diameter of the ileostomy lumen.

The patient with cirrhosis is being taught self-care. Which statement indicates the patient needs more teaching? a. "If I notice a fast heart rate or irregular beats, this is normal for cirrhosis." b. "I need to take good care of my belly and ankle skin where it is swollen." c. "A scrotal support may be more comfortable when I have scrotal edema." d. "I can use pillows to support my head to help me breathe when I am in bed."

A "If I notice a fast heart rate or irregular beats, this is normal for cirrhosis." If the patient with cirrhosis experiences a fast or irregular heart rate, it may be indicative of hypokalemia and should be reported to the health care provider because this is not normal for cirrhosis. Edematous tissue is subject to breakdown and needs meticulous skin care. Pillows and a semi-Fowler's or Fowler's position increase respiratory efficiency. A scrotal support may improve comfort if there is scrotal edema.

The nurse is preparing to administer famotidine to a postoperative patient with a colostomy. The patient states they do not have heartburn. What response by the nurse would be the most appropriate? a. "It will reduce the amount of acid in the stomach." b. "It will prevent air from accumulating in the stomach, causing gas pains." c. "It will prevent the heartburn that occurs as a side effect of general anesthesia." d. "The stress of surgery is likely to cause stomach bleeding if you do not receive it."

A "It will reduce the amount of acid in the stomach." Famotidine is an H2-receptor antagonist that inhibits gastric HCl secretion and thus minimizes damage to gastric mucosa while the patient is not eating a regular diet after surgery. Famotidine does not prevent air from accumulating in the stomach or stop the stomach from bleeding. Heartburn is not a side effect of general anesthesia.

A 50-yr-old patient is preparing to begin breast cancer treatment with tamoxifen. What point should the nurse emphasize when teaching the patient about her new drug regimen? A. "Report any changes in your vision." B. "The medication may cause some breast sensitivity." C. "The drug often alleviates some menopausal symptoms." D. "Abstain from drinking alcohol after you begin taking tamoxifen."

A "Report any changes in your vision." Tamoxifen has the potential to cause cataracts and retinopathy. The drug is likely to exacerbate rather than alleviate perimenopausal symptoms. Breast tenderness is not associated with tamoxifen, and it is not necessary to abstain from alcohol.

The nurse is preparing to insert a nasogastric (NG) tube into a patient with a suspected small intestinal obstruction that is vomiting. The patient asks the nurse why this procedure is necessary. What response by the nurse is most appropriate? a. "The tube will help to drain the stomach contents and prevent further vomiting." b. "The tube will push past the area that is blocked and thus help to stop the vomiting." c. "The tube is just a standard procedure before many types of surgery to the abdomen." d. "The tube will let us measure your stomach contents so we can give you the right IV fluid replacement."

A "The tube will help to drain the stomach contents and prevent further vomiting." The NG tube is used to decompress the stomach by draining stomach contents and thereby prevent further vomiting. The NG tube will not push past the blocked area. Potential surgery is not currently indicated. The location of the obstruction will determine the type of fluid to use, not measure the amount of stomach contents.

The nurse is caring for a woman recently diagnosed with viral hepatitis A. Which individual should the nurse refer for an immunoglobulin (IG) injection? a. A caregiver who lives in the same household with the patient b. A friend who delivers meals to the patient and family each week c. A relative with a history of hepatitis A who visits the patient daily d. A child living in the home who received the hepatitis A vaccine 3 months ago

A A caregiver who lives in the same household with the patient IG is recommended for persons who do not have anti-HAV antibodies and are exposed as a result of close contact with persons who have HAV or foodborne exposure. Persons who have received a dose of HAV vaccine more than 1 month previously or who have a history of HAV infection do not require IG.

An 18-yr-old man who fell through the ice on a pond near his farm was admitted to the emergency department with somnolence. Vital signs are blood pressure of 82 mm Hg systolic with Doppler, respirations of 9 breaths/min, and core temperature of 90°F (32.2°C). The nurse should anticipate which intervention? A. Active core rewarming B. Immersion in a hot bath C. Rehydration and massage D. Passive external rewarming

A Active core rewarming Active internal or core rewarming is used for moderate to severe hypothermia and involves the application of heat directly to the core. Immersion in a hot bath, rehydration, and massage are not appropriate interventions in the treatment of severe hypothermia. Passive rewarming is used in mild hypothermia.

A 47-yr-old man who was lost in the mountains for 2 days is admitted to the emergency department with cold exposure and a core body temperature of 86.6ºF (30.3ºC). Which nursing action is most important? A. Administer warmed IV fluids. B. Position patient under a radiant heat lamp. C. Place an air-filled warming blanket on the patient. D. Immerse the extremities in a water bath (102° to 108°F [38.9° to 42.2°C]).

A Administer warmed IV fluids. A patient with a core body temperature of 86.6ºF (30.3ºC) has moderate hypothermia. Active core rewarming is used for moderate to severe hypothermia and includes administration of warmed IV fluids (109.4ºF [43ºC]). Patients with moderate to severe hypothermia should have the core warmed before the extremities to prevent after drop (or further drop in core temperature). This occurs when cold peripheral blood returns to the central circulation. Use passive or active external rewarming for mild hypothermia. Active external rewarming involves fluid-filled warming blankets or radiant heat lamps. Immersion of extremities in a water bath is indicated for frostbite.

A young male patient is seeking treatment for recurrence of genital tingling, burning, and itching. The nurse will expect a prescription for which class of medications? A. Antivirals B. Antibiotics C. Vaccination D. Contraceptives

A Antivirals This patient is experiencing a reoccurrence of genital herpes (HSV2). Although not a cure, he will be treated with antiviral medications to decrease the duration of viral shedding and the healing time of genital lesions and reduce outbreaks. Antibiotics and contraceptives are not used to treat acute HSV2. There are no vaccinations for HSV.

Which assessment parameter will the nurse address during the secondary survey of a patient in triage? A. Blood pressure and heart rate B. Patency of the patient's airway C. Neurologic status and level of consciousness D. Presence or absence of breath sound and quality of breathing

A Blood pressure and heart rate Vital signs are considered to be a part of the secondary survey in the triage process. Airway, breathing, circulation, and a brief neurologic assessment are components of the primary survey that is done to identify life-threatening conditions.

A patient comes to the clinic after being informed by a sexual partner of possible recent exposure to syphilis. The nurse will examine the patient for what characteristic finding of syphilis in the primary clinical stage? A. Chancre B. Alopecia C. Condylomata lata D. Regional adenopathy

A Chancre Chancres appear in the primary stage of the bacterial invasion of Treponema pallidum, the causative organism of syphilis. The other findings do not appear until the secondary stage of syphilis, occurring a few weeks after the chancres appear.

A patient after a stroke who primarily uses a wheelchair for mobility has developed diarrhea with fecal incontinence. What is a priority assessment by the nurse? a. Fecal impaction b. Perineal hygiene c. Dietary fiber intake d. Antidiarrheal agent use

A Fecal impaction Patients with limited mobility are at risk for fecal impactions caused by constipation that may lead to liquid stool leaking around the hardened impacted feces, so assessing for fecal impaction is the priority. Perineal hygiene can be assessed at the same time. Assessing the dietary fiber and fluid intake and antidiarrheal agent use will be assessed and considered next.

Two days after a colectomy for an abdominal mass, a patient reports gas pains and abdominal distention. The nurse plans care for the patient based on the knowledge that the symptoms are occurring as a result what event? a. Impaired peristalsis b. Irritation of the bowel c. Nasogastric suctioning d. Inflammation of the incision site

A Impaired peristalsis Until peristalsis returns to normal after anesthesia, the patient may experience slowed gastrointestinal motility, leading to gas pains and abdominal distention. Irritation of the bowel, nasogastric suctioning, and inflammation of the surgical site do not cause gas pains or abdominal distention.IS A

A 72-yr-old patient had a mastectomy for breast cancer 6 months ago and wants to have breast reconstructive surgery. Which motivation for surgery would be most likely? A. Improve the woman's self-image B. Be able to experience sexual arousal C. To get a tummy tuck as well as the breast mound D. Restore the premastectomy appearance of the breast

A Improve the woman's self-image The most likely motivation for this patient to seek breast reconstructive surgery is to improve her self-esteem. With this surgery, she will not be able to experience sexual arousal through breast stimulation or restore the premastectomy appearance of the breast. The abdominoplasty (tummy tuck) effect will only be a possibility with the transverse rectus abdominis musculocutaneous (TRAM) flap, not with a breast implant or tissue expansion.

The nurse is conducting discharge teaching for a patient with metastatic lung cancer who was admitted with a bowel impaction. Which instructions would be most helpful to prevent further episodes of constipation? a. Maintain a high intake of fluid and fiber in the diet. b. Discontinue intake of medications causing constipation. c. Eat several small meals per day to maintain bowel motility. d. Sit upright during meals to increase bowel motility by gravity.

A Maintain a high intake of fluid and fiber in the diet. Increased fluid intake and a high-fiber diet reduce the incidence of constipation caused by immobility, medications, and other factors. Fluid and fiber provide bulk that in turn increases peristalsis and bowel motility. Analgesics taken for lung cancer probably cannot be discontinued. Eating several small meals per day and position do not facilitate bowel motility.

A patient with type 2 diabetes and cirrhosis asks the nurse if it would be acceptable to take silymarin (milk thistle) to help minimize liver damage. The nurse responds based on what knowledge? a. Milk thistle may affect liver enzymes and thus alter drug metabolism. b. Milk thistle is generally safe in recommended doses for up to 10 years. c. There is unclear scientific evidence for the use of milk thistle in treating cirrhosis. d. Milk thistle may elevate the serum glucose levels and is thus contraindicated in diabetes.

A Milk thistle may affect liver enzymes and thus alter drug metabolism. Scientific evidence indicates there is no real benefit from milk thistle to protect liver cells from toxic damage in the treatment of cirrhosis. Milk thistle does affect liver enzymes and thus could alter drug metabolism. Therefore, patients will need to be monitored for drug interactions. It is noted to be safe for up to 6 years, not 10 years, and it may lower, not elevate, blood glucose levels.

The nurse is caring for a 25-yr-old patient who has polycystic ovary syndrome (PCOS). When preparing the teaching plan, which classic manifestation should the nurse associate with severity of symptoms and infertility? A. Obesity B. Hirsutism C. Amenorrhea D. Irregular menstrual periods

A Obesity Obesity has been associated with the severity of symptoms such as excess androgens, oligorrhea, amenorrhea, and infertility. This knowledge will affect the teaching the nurse does for this patient to prevent cardiovascular disease and abnormal insulin resistance. Hirsutism, amenorrhea, and irregular menstrual periods are not associated with the severity of the symptoms.

Which guideline for the assessment of intimate partner violence (IPV) should the emergency nurse follow? A. Patients should be routinely screened for family and IPV. B. Patients whom the nurse deems high risk should be assessed for IPV. C. All female patients and patients under 18 should be assessed for IPV. D. Patients should be assessed for IPV provided corroborating evidence exists.

A Patients should be routinely screened for family and IPV. In the emergency department, the nurse needs to screen for family and IPV. Routine screening for this risk factor is required. Such assessment should not be limited to female, high-risk, or young patients, and evidence need not be present to screen for the problem.

Three days after experiencing a series of tick bites, a patient presents to the emergency department. Which manifestation would indicate the patient is experiencing tick paralysis? A. Respiratory distress B. Aggression and frequent falls C. Decreased level of consciousness D. Fever and necrosis at the bite sites

A Respiratory distress A classic manifestation of tick paralysis is flaccid ascending paralysis, which develops over 1 to 2 days. Without tick removal, the patient dies as respiratory muscles become paralyzed. Aggression, decreased level of consciousness, fever, and necrosis at the bite sites are not characteristic of the problem.

When caring for a patient in acute septic shock, what should the nurse anticipate? a. Infusing large amounts of IV fluids b. Administering osmotic and/or loop diuretics c. Administering IV diphenhydramine (Benadryl) d. Assisting with insertion of a ventricular assist device (VAD)

A Septic shock is characterized by a decreased circulating blood volume. Volume expansion with the administration of IV fluids is the cornerstone of therapy.

The wound, ostomy, and continence nurse (WOCN) selects the site where the ostomy will be placed. What should be included in site consideration? a. The patient must be able to see the site. b. The site should be outside the rectus muscle area. c. It is easier to seal the drainage bag to a protruding area. d. A waistline site will allow using a belt to hold the appliance in place.

A The patient must be able to see the site. In selection of the ostomy site, the WOC nurse will want a site visible to the patient so the patient can take care of it, within the rectus muscle to avoid hernias, and on a flat surface to more easily create a good seal with the drainage bag. Care should be taken to avoid skin creases, scars, and belt lines, which can interfere with the adherence of the appliance.

A patient presents with complaints of mastalgia. After determining cancer is not present, which strategies may provide relief (select all that apply.)? A. Application of ice B. Oral contraceptives C. Reduce caffeine intake D. Increase intake of fluids E. Oral antibiotic administration F. Wear a supportive bra during the day

A, B, C To reduce breast pain, the patient may apply ice, reduce caffeine intake, and be considered for oral contraceptive therapy. Increasing fluids and use of antibiotics are not indicated for treatment of mastalgia. Wearing a supportive bra continuously may reduce discomfort.

When caring for a patient with a biliary obstruction, the nurse will anticipate administering which vitamin supplements (select all that apply.)? a. Vitamin A b. Vitamin D c. Vitamin E d. Vitamin K e. Vitamin B

A, B, C, D Biliary obstruction prevents bile from entering the small intestine and thus prevents the absorption of fat-soluble vitamins. Vitamins A, D, E, and K are all fat soluble and thus would need to be supplemented in a patient with biliary obstruction.

A hospitalized patient has just been diagnosed with diarrhea due to Clostridium difficile. Which nursing interventions should be included in the patient's plan of care (select all that apply.)? a. Initiate contact isolation precautions. b. Place the patient on a clear liquid diet. c. Disinfect the room with 10% bleach solution. d. Teach any visitors to wear gloves and gowns. e. Use hand sanitizer before and after patient or bodily fluid contact.

A, C, D Initiation of contact isolation precautions must be done immediately with a patient with C. difficile, which includes washing hands with soap and water before and after patient or bodily fluid contact. Alcohol-based sanitizers are ineffective. Visitors need to be taught to wear gloves and gowns and wash hands. A clear liquid diet is not necessary. The room will be disinfected with 10% bleach solution when the patient is dismissed and may be done periodically during the patient's stay, depending on the agency policy.

What should teaching for patients with a sexually transmitted infection (STI) include (select all that apply.)? A. Treatment of sexual partners B. Douching may help to provide relief of itching. C. Importance of retesting after treatment to confirm cure D. Cotton undergarments are preferred over synthetic materials. E. Sexual abstinence is indicated during the communicable phase of the disease. F. Condoms should be used during as well as after treatment during sexual activity.

A, C, D, E, F Teaching for patients with an STI should include the treatment of all sexual partners, retesting after treatment to confirm cure, cotton undergarments are more comfortable, sexual abstinence is needed during the communicable phase of the infection to prevent spread, and condoms should be used for sexual activity during and after treatment to prevent spread and reinfection. Douching may spread the infection or alter the local immune responses and is therefore contraindicated in patients with STIs.

The nurse is volunteering at a community center to teach women about breast cancer. What should the nurse include when discussing risk factors (select all that apply.)? A. Nulliparity B. Age 30 or over C. Early menarche D. Late menopause E. Alcohol consumption F. Personal history of colon cancer

A, C, D, E, F Women are at an increased risk for development of breast cancer if they are older than the age of 50 years; have a family history of breast cancer; have a personal history of breast, colon, endometrial, or ovarian cancer; have a long menstrual history as seen with early menarche or late menopause; and have had a first full-term pregnancy after the age of 30 years or are nulliparous. Alcohol consumption may increase the risk of breast cancer.

The patient has been part of a community emergency response team (CERT) for a tropical storm in Dallas with temperatures near 100°F (37.7°C) for the past 2 weeks. When assessing the patient, the nurse finds hypotension, body temperature of 104°F (40°C), dry and ashen skin, and neurologic symptoms. What treatments should the nurse anticipate (select all that apply.)? A. Administer 100% O2. B. Immerse in an ice bath. C. Administer cool IV fluids. D. Cover the patient to prevent chilling. E. Administer acetaminophen (Tylenol). F. Administer chlorpromazine for shivering.

A, C, F The patient is experiencing heatstroke. Treatment focuses first on stabilizing the patient's ABC and rapidly reducing the core temperature. Administration of 100% O2 compensates for the patient's hypermetabolic state. Cooling the body with IV fluids is effective. Immersion in an ice bath will cause shivers that increase core temperature, so a cool water bath should be used for conductive cooling. Removing the clothing, covering the patient with wet sheets, and placing the patient in front of a fan will cause evaporative cooling. If shivering ensues, treatment with chlorpromazine is indicated. Shivering increases core temperature due to the heat generated by muscle activity. Excessive covers will not be used. Acetaminophen will not be effective because the increase in temperature is not related to infection.

The nurse is providing teaching to a group of perimenopausal women. Which herbs and/or supplements would the nurse include in a discussion regarding effective alternative therapies for menopausal symptoms (select all that apply.)? A. Soy B. Garlic C. Gingko D. Vitamin A E. Cinnamon F. Black cohosh

A, F There is good scientific evidence that soy is useful in decreasing menopausal hot flashes and that black cohosh is safe to use for up to 6 months to decrease menopausal symptoms. Garlic, gingko, vitamin A, and cinnamon do not affect menopausal symptoms.

A 50-yr-old woman with a suspected brain tumor is scheduled for a CT scan with contrast media. The nurse notifies the physician that the patient reported an allergy to shellfish. Which response by the physician should the nurse question? a. Infuse IV diphenhydramine before the procedure. b. Administer lorazepam (Ativan) before the procedure. c. Complete the CT scan without the use of contrast media. d. Premedicate with hydrocortisone sodium succinate (Solu-Cortef).

B

A 64-yr-old woman is admitted to the emergency department vomiting bright red blood. The patient's vital signs are blood pressure of 78/58 mm Hg, pulse of 124 beats/min, respirations of 28 breaths/min, and temperature of 97.2°F (36.2°C). Which physician order should the nurse complete first? a. Obtain a 12-lead ECG and arterial blood gases b. Rapidly administer 1000 mL normal saline solution IV. c. Administer norepinephrine (Levophed) by continuous IV infusion. d. Carefully insert a nasogastric tube and an indwelling bladder catheter.

B

A 71-yr-old woman arrives in the emergency department after ingesting 8 g of acetaminophen (Tylenol). Which question is most important for the nurse to ask? A. "Do you feel like you have a fever?" B. "What time did you take the medication?" C. "Have you tried to commit suicide before?" D. "Are you experiencing any abdominal pain?"

B "What time did you take the medication?" Acetaminophen will bind to activated charcoal and pass through the gastrointestinal tract without being absorbed. Activated charcoal is most effective if administered within 1 hour of ingestion of acetaminophen and other select poisons.

A 22-yr-old man is being treated at a college health care clinic for gonorrhea. What should the nurse include in patient teaching? A. "While being treated for the infection, you will not be able to pass this infection on to your sexual partner." B. "While you're taking the antibiotics, you will need to abstain from participating in sexual activity and drinking alcohol." C. "It's important to complete your full course of antibiotics in order to ensure that you become resistant to reinfection." D. "The symptoms of gonorrhea will resolve on their own, but it is important for you to abstain from sexual activity while this is occurring."

B "While you're taking the antibiotics, you will need to abstain from participating in sexual activity and drinking alcohol." Treatment for gonorrhea necessitates abstinence from sexual activity (to prevent infection of partners) and alcohol (to avoid urethral irritation). The disease is not self-limiting, nor does successful treatment confer future resistance.

A patient is given a bisacodyl suppository and asks the nurse how long it will take to work. What is the best response by the nurse? a. 2 to 5 minutes b. 15 to 60 minutes c. 2 to 4 hours d. 6 to 8 hours

B 15 to 60 minutes Bisacodyl suppositories usually are effective within 15 to 60 minutes of administration, so the nurse should plan accordingly to assist the patient to use the bedpan or commode.

A patient with suspected bowel obstruction had a nasogastric tube inserted at 4:00 AM. The nurse shares in the morning report that the day shift staff should check the tube for patency at what times? a. 7:00 AM, 10:00 AM, and 1:00 PM b. 8:00 AM, 12:00 PM, and 4:00 PM c. 9:00 AM and 3:00 PM d. 9:00 AM, 12:00 PM, and 3:00 PM

B 8:00 AM, 12:00 PM, and 4:00 PM A nasogastric tube should be checked for patency routinely at 4-hour intervals. Thus if the tube were inserted at 4:00 AM, it would be due to be checked at 8:00 AM, 12:00 PM, and 4:00 PM.

A nurse is performing triage in the emergency department. Which patient should the nurse see first? A. A 18-yr-old patient with type 1 diabetes mellitus who has a 4-cm laceration on right leg B. A 32-yr-old patient with drug overdose who is unresponsive with a poor respiratory effort C. A 56-yr-old patient with substernal chest pain who is diaphoretic with shortness of breath D. A 78-yr-old patient with right hip fracture who is confused; blood pressure is 98/62 mm Hg

B A 32-yr-old patient with drug overdose who is unresponsive with a poor respiratory effort The patient with a drug overdose is unstable and needs to be seen immediately. Patient with chest pain (possible myocardial infarction) should be seen second. Patient with hip fracture should be seen third. Patient with laceration is the most stable and should be seen last.

The nurse identifies that which patient is at highest risk for developing colon cancer? a. A 28-yr-old man who has a body mass index of 27 kg/m2 b. A 32-yr-old woman with a 12-year history of ulcerative colitis c. A 52-yr-old man who has followed a vegetarian diet for 24 years d. A 58-yr-old woman taking prescribed estrogen replacement therapy

B A 32-yr-old woman with a 12-year history of ulcerative colitis Risk for colon cancer includes personal history of inflammatory bowel disease (especially ulcerative colitis for longer than 10 years); obesity (body mass index ?5= 30 kg/m2); family (first-degree relative) or personal history of colorectal cancer, adenomatous polyposis, or hereditary nonpolyposis colorectal cancer syndrome; eating red meat (?5=7 servings/week); cigarette use; and drinking alcohol (?5=4 drinks/week).

The nurse is caring for a group of patients. Which patient has the highest risk for developing pancreatic cancer? a. A 38-yr-old Hispanic woman who is obese and has hyperinsulinemia b. A 72-yr-old African American man who has smoked cigarettes for 50 years c. A 23-yr-old man who has cystic fibrosis-related pancreatic enzyme insufficiency d. A 19-yr-old patient who has a 5-year history of uncontrolled type 1 diabetes mellitus

B A 72-yr-old African American man who has smoked cigarettes for 50 years Risk factors for pancreatic cancer include chronic pancreatitis, diabetes mellitus, age, cigarette smoking, family history of pancreatic cancer, high-fat diet, and exposure to chemicals such as benzidine. African Americans have a higher incidence of pancreatic cancer than whites. The most firmly established environmental risk factor is cigarette smoking. Smokers are two or three times more likely to develop pancreatic cancer compared with nonsmokers. The risk is related to duration and number of cigarettes smoked.

When providing discharge teaching for a patient after a laparoscopic cholecystectomy, what information should the nurse include? a. Do not return to work or normal activities for 3 weeks. b. A lower-fat diet may be better tolerated for several weeks. c. Bile-colored drainage will probably drain from the incision. d. Keep the bandages on and the puncture site dry until it heals.

B A lower-fat diet may be better tolerated for several weeks. Although the usual diet can be resumed, a low-fat diet is usually better tolerated for several weeks after surgery. Normal activities can be gradually resumed as the patient tolerates. Bile-colored drainage or pus, redness, swelling, severe pain, and fever may all indicate infection. The bandage may be removed the day after surgery, and the patient can shower.

A 52-yr-old man with a primary infection of genital herpes was prescribed acyclovir (Zovirax) orally for 10 days. The patient returns to the clinic for a follow-up visit. Which finding indicates that treatment is effective? A. Negative bacterial culture B. Absence of genital lesions C. Reduction of genital warts D. No drainage from chancre sore

B Absence of genital lesions Primary genital herpes is a viral disorder caused by the herpes simplex virus. Genital herpes results in painful, vesicular lesions. The lesions rupture, form crusts, and heal in 17 to 21 days. Genital warts are caused by the human papillomavirus. Genital herpes is caused by a viral infection (not bacterial). Syphilis is caused by a bacterial organism and results in a chancre, which is a painless, indurated lesion.

The patient is being treated for a recurrent episode of Chlamydia. What should the nurse include in patient teaching? A. If you are treated, your sexual partner will not need to be treated. B. Abstain from sexual intercourse for 7 days after finishing the treatment. C. You will probably get gonorrhea if you have another recurrence of Chlamydia. D. Because you have been treated before, you do not need to take a full course of medication this time.

B Abstain from sexual intercourse for 7 days after finishing the treatment. Patients treated for Chlamydia infections should abstain from sexual intercourse for 7 days after treatment until all sexual partners have completed a full course of treatment to prevent recurrence, and a follow-up culture is done. A single-dose treatment is also available. Because Chlamydia and gonococcal infections are closely associated, they are frequently both treated, but having Chlamydia does not give the patient gonorrhea. Each time a patient is treated for Chlamydia, a full course of treatment is required.

A colectomy is scheduled for a patient with ulcerative colitis. The nurse should plan to include which prescribed measure in the preoperative preparation of this patient? a. Instruction on irrigating a colostomy b. Administration of a cleansing enema c. A high-fiber diet the day before surgery d. Administration of IV antibiotics for bowel preparation

B Administration of a cleansing enema Preoperative preparation for bowel surgery typically includes bowel cleansing with antibiotics, such as oral neomycin and cleansing enemas, including Fleet enemas. Instructions to irrigate the colostomy will be done postoperatively. Oral antibiotics are given preoperatively and an IV antibiotic may be used in the operating room. A clear liquid diet will be used the day before surgery with the bowel cleansing.

A 24-yr-old patient had breast augmentation surgery and will be discharged the same day. What instructions should the nurse provide to minimize the risk of complications in the immediate recovery period? A. Avoid wearing a bra until postoperative day 3. B. Ask the patient to avoid strenuous exercise during her recovery period. C. Sleep in a semi-Fowler's position until her scheduled follow-up appointment. D. Enlist a friend or family member to perform passive range-of-motion exercises.

B Ask the patient to avoid strenuous exercise during her recovery period. As with all types of breast surgery, strenuous exercise is contraindicated during the recovery period after breast augmentation. A bra should be worn to prevent dehiscence and provide comfort. Sleeping in a semi-Fowler's position is not necessary, and passive range-of-motion exercises should be avoided at first.

A 23-yr-old woman admitted with a possible ectopic tubal pregnancy reports sudden intense pelvic pain radiating to the left shoulder. Which action by the nurse should receive the highest priority? A. Observe the amount of vaginal bleeding every 15 minutes for 1 hour. B. Check the vital signs and immediately notify the health care provider. C. Administer the prescribed pain medication and reassess in 30 minutes. D. Assess the fetal heart tones and determine the presence of fetal movement.

B Check the vital signs and immediately notify the health care provider. A ruptured ectopic pregnancy may produce pelvic or abdominal pain and vaginal bleeding. If the tube ruptures, the pain is intense and may be referred to the shoulder. External vaginal bleeding may not be an accurate indicator of actual blood loss. Vital signs should be monitored closely along with observation for signs of shock. A ruptured ectopic pregnancy is an emergency because of the risk of hemorrhage and hypovolemic shock. The patient may need a blood transfusion and IV fluid therapy. In addition, the patient will need emergency surgery. Fetal assessment is not indicated for an ectopic pregnancy.

When caring for a critically ill patient who is being mechanically ventilated, the nurse will monitor for which clinical manifestation of multiple organ dysfunction syndrome (MODS)? a. Increased serum albumin b. Decreased respiratory compliance c. Increased gastrointestinal (GI) motility d. Decreased blood urea nitrogen (BUN)/creatinine ratio

B Clinical manifestations of MODS include symptoms of respiratory distress, signs and symptoms of decreased renal perfusion, decreased serum albumin and prealbumin, decreased GI motility, acute neurologic changes, myocardial dysfunction, disseminated intravascular coagulation (DIC), and changes in glucose metabolism.

The patient with right upper quadrant abdominal pain has an abdominal ultrasound that reveals cholelithiasis. What is the nurse's priority? a. Prevent all oral intake. b. Control abdominal pain. c. Provide enteral feedings. d. Avoid dietary cholesterol.

B Control abdominal pain. Patients with cholelithiasis can have severe pain, so controlling pain is important until the problem can be treated. NPO status may be needed if the patient will have surgery but will not be used for all patients with cholelithiasis. Patients with pancreatitis may be NPO. Enteral feedings should not be needed, and avoiding dietary cholesterol is not used to treat cholelithiasis.

The nurse is teaching health promotion to a variety of women in a community center. When asked when a female should begin having a Pap smear, how should the nurse respond? A. Every year beginning at age 30 years B. Every 3 years beginning at age 21 years C. Every 3 years beginning at age 18 years if sexually active D. Every year beginning at the onset of menarche and continuing until menopause

B Every 3 years beginning at age 21 years A Pap test (Pap smear) should be done at least once every 3 years at the age of 21 years regardless of when a woman becomes sexually active. Women 65 years or older may stop having Pap tests after having no abnormal Pap test results for the previous 2 years.

The nurse is developing a plan of care for a patient with an abdominal mass and suspected bowel obstruction. Which factor in the patient's history does the nurse recognize as increasing the patient's risk for colorectal cancer? a. Osteoarthritis b. History of colorectal polyps c. History of lactose intolerance d. Use of herbs as dietary supplements

B History of colorectal polyps A history of colorectal polyps places this patient at risk for colorectal cancer. This tissue can degenerate over time and become malignant. Osteoarthritis, lactose intolerance, and the use of herbs do not pose additional risk to the patient.

What information would have the highest priority for the nurse to include in preoperative teaching for a patient scheduled for a colectomy? a. How to care for the wound b. How to deep breathe and cough c. The location and care of drains after surgery d. Which medications will be used during surgery

B How to deep breathe and cough Because anesthesia, an abdominal incision, and pain can impair the patient's respiratory status in the postoperative period, it is of high priority to teach the patient to cough and deep breathe. Otherwise, the patient could develop atelectasis and pneumonia, which would delay early recovery from surgery and hospital discharge. Care for the wound and location and care of the drains will be briefly discussed preoperatively but will be done again with higher priority after surgery. Knowing which drugs will be used during surgery may not be meaningful to the patient and should be reviewed with the patient by the anesthesiologist.

The nurse would recognize which clinical manifestation as suggestive of sepsis? a. Sudden diuresis unrelated to drug therapy b. Hyperglycemia in the absence of diabetes c. Respiratory rate of seven breaths per minute d. Bradycardia with sudden increase in blood pressure

B Hyperglycemia in patients with no history of diabetes is a diagnostic criterion for sepsis. Oliguria, not diuresis, typically accompanies sepsis along with tachypnea and tachycardia.

A nurse in the emergency department is working triage. Which patient assessment findings would indicate immediate care is required? A. Shortened and externally rotated leg B. Inability to swallow and move the left arm C. Warm, edematous, reddened and painful calf D. Yellow sputum and pain with deep inspiration

B Inability to swallow and move the left arm Inability to swallow and move the left arm suggests the patient is experiencing a stroke. A CT scan is indicated within 25 minutes of arrival to determine ischemic versus hemorrhagic origin, which will delineate available treatments. The warm, edematous, reddened and painful calf suggests deep vein thrombosis. Although not an immediate threat, there is a risk of pulmonary emboli. A shortened and externally rotated leg suggests a hip fracture. A patient with yellow sputum and pain with deep inspiration suggest a pneumonia that may require hospitalization or could be treated as an outpatient.

A patient with sudden pain in the left upper quadrant radiating to the back and vomiting was diagnosed with acute pancreatitis. Which intervention should the nurse include in the patient's plan of care? a. Immediately start enteral feeding to prevent malnutrition. b. Insert an NG and maintain NPO status to allow pancreas to rest. c. Initiate early prophylactic antibiotic therapy to prevent infection. d. Administer acetaminophen (Tylenol) every 4 hours for pain relief.

B Insert an NG and maintain NPO status to allow pancreas to rest. Initial treatment with acute pancreatitis will include an NG tube if there is vomiting and being NPO to decrease pancreatic enzyme stimulation and allow the pancreas to rest and heal. Fluid will be administered to treat or prevent shock. The pain will be treated with IV morphine because of the NPO status. Enteral feedings will only be used for the patient with severe acute pancreatitis in whom oral intake is not resumed. Antibiotic therapy is only needed with acute necrotizing pancreatitis and signs of infection.

In reviewing the chart, which patient assessment is likely to have the greatest impact on this patient's risk of death from the accident? Current assessment: Male found floating face down after surfing accident, CPR done by rescuers, Sinus tachycardia with frequent premature ventricular contractions (PVCs), Mechanical ventilation, Left pupil size 10 cm, not reactive to light, Pulmonary artery wedge pressure (PAWP), 16 mm Hg, PaO2 108 mm Hg, FIO2 50%, PEEP 5 cm, Cool extremities, weak peripheral pulses A. PAWP 16 mm Hg B. Left pupil 10 cm, not reactive to light C. Sinus tachycardia with frequent PVCs D. Cool extremities, weak peripheral pulses

B Left pupil 10 cm, not reactive to light Unilateral pupil dilation without response to light can be a clinical indicator of tentorial herniation of the brain and can occur in a surfing accident as the surfboard and patient are forcefully tossed around in the waves. If the excessive intracranial pressure is allowed to continue, the patient is at a high risk for brainstem death. This finding merits emergency interventions to prevent death. The PAWP, sinus tachycardia with frequent PVCs, and cool extremities with weak peripheral pulses do not indicate imminent death.

The nurse is caring for a 55-yr-old man patient with acute pancreatitis resulting from gallstones. Which clinical manifestation would the nurse expect? a. Hematochezia b. Left upper abdominal pain c. Ascites and peripheral edema d. Temperature over 102o F (38.9o C)

B Left upper abdominal pain Abdominal pain (usually in the left upper quadrant) is the predominant manifestation of acute pancreatitis. Other manifestations of acute pancreatitis include nausea and vomiting, low-grade fever, leukocytosis, hypotension, tachycardia, and jaundice. Abdominal tenderness with muscle guarding is common. Bowel sounds may be decreased or absent. Ileus may occur and causes marked abdominal distention. Areas of cyanosis or greenish to yellow-brown discoloration of the abdominal wall may occur. Other areas of ecchymoses are the flanks (Grey Turner's spots or sign, a bluish flank discoloration) and the periumbilical area (Cullen's sign, a bluish periumbilical discoloration).

What should the nurse emphasize in the teaching for a woman diagnosed with pelvic inflammatory disease (PID)? A. The importance of contraception B. Manifestations of further infection C. The importance of maintaining hygiene D. Benefits of hormone replacement therapy (HRT)

B Manifestations of further infection PID frequently progresses to serious infection of the reproductive structures. The diagnosis does not present a particular need for contraception or specific hygiene measures. HRT is not used to treat PID.

Which task could the registered nurse delegate to unlicensed assistive personnel (UAP) during the care of a patient who has had recent transverse rectus abdominis musculocutaneous (TRAM) flap surgery? A. Document the condition of the patient's incisions. B. Mobilize the patient in a slightly hunched position. C. Change the patient's abdominal and chest dressings. D. Change the parameters of the patient-controlled analgesic (PCA) pump.

B Mobilize the patient in a slightly hunched position. Mobilization of a postsurgical patient may be delegated, and a patient who has had a TRAM flap should not stand or walk fully erect in order to minimize strain on the incisions. Changing dressings, assessing wounds, and reprogramming a PCA pump are not appropriate tasks to delegate to UAP.

The nurse should administer an as-needed dose of magnesium hydroxide after noting what information when reviewing a patient's medical record? a. Abdominal pain and bloating b. No bowel movement for 3 days c. A decrease in appetite by 50% over 24 hours d. Muscle tremors and other signs of hypomagnesemia

B No bowel movement for 3 days Magnesium hydroxide is an osmotic laxative that produces a soft, semisolid stool usually within 15 minutes to 3 hours. This medication would benefit the patient who has not had a bowel movement for 3 days. It would not be given for abdominal pain and bloating, decreased appetite, or signs of hypomagnesemia.

A 54-yr-old patient admitted with diabetes mellitus, malnutrition, osteomyelitis, and alcohol abuse has a serum amylase level of 280 U/L and a serum lipase level of 310 U/L. Which diagnosis does the nurse expect? a. Starvation b. Pancreatitis c. Systemic sepsis d. Diabetic ketoacidosis

B Pancreatitis The patient with alcohol abuse could develop pancreatitis as a complication, which would increase the serum amylase (normal, 30-122 U/L) and serum lipase (normal, 31-186 U/L) levels as shown.

The nurse administers a Gardasil vaccine to an 18-yr-old female patient. After the injection, which patient instruction is priority? A. Avoid sexual activity for 24 to 48 hours. B. Remain lying down for at least 15 minutes. C. Return to the clinic in 6 months for a second dose. D. Use two methods of birth control to avoid pregnancy.

B Remain lying down for at least 15 minutes. To prevent syncope (fainting) during and after the administration of Gardasil, the patient should remain sitting or lying down for 15 minutes. The vaccine is not recommended during pregnancy. Gardasil vaccine is given in three IM doses over a 6-month period. There are no sexual activity restrictions after administration of Gardasil.

A pregnant woman is experiencing amenorrhea, morning sickness, and breast tenderness. In the ninth week after her last menstrual period, she is rushed to the hospital with severe left shoulder pain, blood pressure of 90/60 mm Hg, and heart rate of 112 beats/min. What is the best diagnostic test is expected? A. Serum hemoglobin B. Transvaginal ultrasound C. 12-lead electrocardiogram (ECG) D. Serial ß-human chorionic gonadotropin levels

B Transvaginal ultrasound Because the patient is known to be pregnant, a transvaginal ultrasound will be used to assess for ectopic pregnancy and tubal rupture. Serum hemoglobin and 12-lead ECG would not define a diagnosis related to the manifestations that she has. Serial ß-human chorionic gonadotropin levels could be used if the patient was stable to determine if a spontaneous abortion is occurring because the levels would decrease over time.

The nurse is aware of potential complications related to cirrhosis. Which interventions would be included in a safe plan of care (select all that apply.)? a. Provide a high-protein, low-carbohydrate diet. b. Teach the patient to use soft-bristle toothbrush and electric razor. c. Teach the patient to avoid vigorous blowing of nose and coughing. d. Apply gentle pressure for the shortest possible time after venipuncture. e. Use the smallest gauge needle possible when giving injections or drawing blood. f. Instruct the patient to avoid aspirin and nonsteroidal antiinflammatory (NSAIDs).

B, C, E, F Using the smallest gauge needle for injections, using a soft bristle toothbrush and an electric razor will minimize the risk of bleeding into the tissues. Avoiding straining, nose blowing, and coughing will reduce the risk of hemorrhage at these sites. The nurse should apply gentle but prolonged pressure to venipuncture sites to minimize the risk of bleeding. Aspirin and NSAIDs should not be used in patients with liver disease because they interfere with platelet aggregation, thus increasing the risk for bleeding. A low-salt, low-protein, high-carbohydrate diet may be recommended.

The patient has hypovolemic shock. By compensatory mechanisms, the decreased blood pressure (BP) has led to decreased capillary hydrostatic pressure, activation of the sympathetic nervous system and α- and β-adrenergic stimulation, decreased blood flow, and increased cardiac output (CO). In what order should the next pathophysiologic mechanisms of the compensated stage of hypovolemic shock occur? (Answer with a letter followed by a comma and a space (e.g. A, B, C, D).) a. Increased ADH leads to renal water reabsorption and increased blood volume. b. Decreased kidney perfusion leads to renin released and increased angiotensin I. c. Increased renal sodium reabsorption leads to increased serum osmolality that leads to release of ADH. d. Increased angiotensin II leads to aldosterone secretion and results in increased renal sodium reabsorption. e. Increased angiotensin II leads to venous and arterial vasoconstriction, increased venous return to the heart, and increased BP.

B, D, E, C, A

A female patient is recovering from rectocele repair surgery. Which interventions should be included in the plan of care (select all that apply.)? A. Maintain complete bed rest. B, Administer a stool softener. C. Provide a cleansing enema. D. Apply ice to the perineal area. E. Urinary catheter care twice a day. F. Sitz bath may be used in a few days.

B, D, E, F Administering a stool softener will reduce straining and disruption of the surgical repair. Ice will reduce pain and swelling at the surgical site. Urinary catheter care is provided twice a day to reduce catheter-associated urinary tract infections. A sitz bath may be given a few days after surgery for comfort. Maintaining strict bed rest is not indicated. A cleansing enema is provided before surgery, not after.

A patient with cirrhosis has increased abdominal girth from ascites. Which items identify the pathophysiology related to ascites (select all that apply.)? a. Hepatocytes are unable to convert ammonia to urea. b. Osmoreceptors in the hypothalamus stimulate thirst. c. An enlarged spleen removes blood cells from the circulation. d. Portal hypertension causes leaking of protein and water into the peritoneal cavity. e. Aldosterone is released to stabilize intravascular volume by saving salt and water. f. Inability of the liver to synthesize albumin reducing intravascular oncotic pressure.

B, D, E, F Ascites related to cirrhosis is caused by decreased colloid oncotic pressure. The liver does not produce albumin that holds fluid in the vascular space, so fluid shifts into interstitial and third spaces. Portal hypertension causes back pressure in the vessels, shifting protein and fluids into the peritoneal cavity. Decreased intravascular volume stimulates the release of aldosterone, which increases sodium and fluid retention. Oral intake of fluids and removal of blood cells by the spleen do not directly contribute to ascites.

The patient at the clinic complains of abdominal bloating, depression, and irritability related to premenstrual syndrome. What should the nurse recommend initially (select all that apply.)? A. Take diuretics. B. Exercise regularly. C. Take antidepressants. D. Take antianxiety agents. E. Increase pork, chicken, and milk intake. F. Consider psychological counseling to resolve symptoms.

B, E The nurse can recommend regular exercise to help manage stress, elevate the mood, and have a relaxing effect. Eating foods rich in vitamin B6 (pork, milk, and legumes) and tryptophan (dairy and poultry) will promote serotonin production and improve symptoms. Diuretics, antidepressants, and antianxiety agents are not prescribed unless symptoms persist or interfere with daily functioning. Psychological counseling does not address physiological symptoms, but it may improve coping mechanisms.

The nurse is caring for a 29-yr-old man who was admitted 1 week ago with multiple rib fractures, pulmonary contusions, and a left femur fracture from a motor vehicle crash. The attending physician states the patient has developed sepsis, and the family members have many questions. Which information should the nurse include when explaining the early stage of sepsis? a. Antibiotics are not useful when an infection has progressed to sepsis. b. Weaning the patient away from the ventilator is the top priority in sepsis. c. Large amounts of IV fluid are required in sepsis to fill dilated blood vessels d. The patient has recovered from sepsis if he has warm skin and ruddy cheeks.

C

A 19-yr-old man comes to the outpatient clinic for treatment of uncomplicated gonorrhea. Which patient statement requires immediate clarification by the nurse? A. "I should avoid alcohol intake for at least 2 weeks." B. "I will have my sexual partner come in for treatment." C. "After I start the antibiotic, it is safe to have sex again." D. "After the treatment, I do not need to return to the clinic for retesting."

C "After I start the antibiotic, it is safe to have sex again." Patients should avoid sexual intercourse for 7 days after completing treatment with antibiotics. All sexual contact of patients with gonorrhea must be evaluated and treated to prevent reinfection. Patients should abstain from sexual intercourse and alcohol during treatment. Sexual intercourse allows the infection to spread and can delay healing. Alcohol is irritating to the healing urethral walls. Patients with uncomplicated gonorrhea who are treated do not need to return to the clinic to confirm the disease has been cured.

The nurse provides discharge instructions for a 64-yr-old woman with ascites and peripheral edema related to cirrhosis. Which patient statement indicates teaching was effective? a. "Lactulose should be taken every day to prevent constipation." b. "It is safe to take acetaminophen up to four times a day for pain." c. "Herbs and other spices should be used to season my foods instead of salt." d. "I will eat foods high in potassium while taking spironolactone (Aldactone)."

C "Herbs and other spices should be used to season my foods instead of salt." A low-sodium diet is indicated for patients with ascites and edema related to cirrhosis. Table salt is a well-known source of sodium and should be avoided. Alternatives to salt to season foods include the use of seasonings such as garlic, parsley, onion, lemon juice, and spices. Pain medications such as acetaminophen, aspirin, and ibuprofen should be avoided because these medications may be toxic to the liver. The patient should avoid potentially hepatotoxic over-the-counter drugs (e.g., acetaminophen) because the diseased liver is unable to metabolize these drugs. Spironolactone is a potassium-sparing diuretic. Lactulose results in the acidification of feces in bowel and trapping of ammonia, causing its elimination in feces.

The nurse provides medication instruction for a 30-yr-old woman who is prescribed clomiphene (Clomid). Which patient statement is most important for the nurse to clarify? A. "Hormone production and release will be increased." B. "This drug is like estrogen and is used to treat infertility." C. "I should avoid intercourse while taking this medication." D. "This medication will stimulate my ovaries to produce eggs."

C "I should avoid intercourse while taking this medication." Clomiphene is an oral medication administered for infertility. The medication is a selective estrogen-stimulation modulator that stimulates ovulation, making pregnancy after intercourse or artificial insemination more likely. The drug increases gonadotropin-releasing hormone production. In addition, the release of the follicle-stimulating hormone and luteinizing hormone is increased.

The nurse is caring for an obese 67-yr-old woman after a right mastectomy with axillary lymph node dissection. Which discharge instruction should the nurse include? A. "Arm exercises should not be started for 4 to 6 weeks." B. "Discontinue arm exercises if you have discomfort or pain." C. "Special massage therapy can decrease swelling in your arm." D. "Keep your right arm in a sling to decrease pain and swelling."

C "Special massage therapy can decrease swelling in your arm." Decongestive therapy may be used for acute lymphedema and includes a massage-like technique to mobilize the subcutaneous accumulation of fluid. Arm exercises should be performed to prevent contractures and muscle shortening, maintain muscle tone, and improve lymph and blood circulation. The arm exercises should be initiated after surgery and increased gradually. Pain medications should be administered 30 minutes before arm exercises. The operative arm should be kept at the level of the heart but not in a sling; a sling discourages use of the arm.

The nurse instructs a 50-yr-old woman about cholestyramine to reduce pruritus caused by gallbladder disease. Which patient statement indicates understanding of the instructions? a. "This medication will help me digest fats and fat-soluble vitamins." b. "I will apply the medicated lotion sparingly to the areas where I itch." c. "The medication is a powder and needs to be mixed with milk or juice." d. "I should take this medication on an empty stomach at the same time each day."

C "The medication is a powder and needs to be mixed with milk or juice." For treatment of pruritus, cholestyramine may provide relief. This is a resin that binds bile salts in the intestine, increasing their excretion in the feces. Cholestyramine is in powder form and should be mixed with milk or juice before oral administration.

The nurse caring for patients in a primary care clinic identifies which patient as being the most at risk for the development of breast cancer? A. A 25-yr-old female patient with fibrocystic breast disease B. A 59-yr-old male patient who has inherited the APC gene C. A 72-yr-old female patient with a family history of breast cancer D. A 43-yr-old male patient who is obese and leads a sedentary lifestyle

C A 72-yr-old female patient with a family history of breast cancer The risk factors most associated with breast cancer are female gender, advancing age, and family history. The incidence of breast cancer increases dramatically after age 60 years. Mutations in BRCA genes may cause 5% to 10% of breast cancers; The APC gene is associated with colon cancer. Obesity and physical inactivity increase the risk for breast cancer. Fibrocystic breast disease is not associated with the development of breast cancer.

A patient is planned for discharge home today after ostomy surgery for colon cancer. The nurse should assign the patient to which staff member? a. A nursing assistant on the unit who also has hospice experience b. A licensed practical nurse that has worked on the unit for 10 years c. A registered nurse with 6 months of experience on the surgical unit d. A registered nurse who has floated to the surgical unit from pediatrics

C A registered nurse with 6 months of experience on the surgical unit The patient needs ostomy care directions and reinforcement at discharge and should be assigned to a registered nurse with experience in providing discharge teaching for ostomy care. Teaching should not be delegated to a licensed practical/vocational nurse or unlicensed assistive personnel.

In working with teenagers, what should the nurse include when teaching about prevention of STIs? A. Spermicidal jellies reduce the risk of getting STIs. B. STIs are easily cured so prevention is not important. C. Abstinence and then condoms are the best prevention. D. Douches for women and cleaning the penis will prevent STIs.

C Abstinence and then condoms are the best prevention. Abstinence and then condom use are the best prevention of STIs. Spermicidal jellies or creams do not reduce the risk of contracting STIs. Most STIs are curable, but complications are serious and costly if they are not cured. Douches may spread the infection, undermine local immune responses, and do not prevent STIs. Cleansing of the penis will provide comfort after an STI has been diagnosed but will not prevent STIs.

An 18-yr-old young woman has been admitted to the emergency department after ingesting an entire bottle of chewable multivitamins in a suicide attempt. The nurse should anticipate which intervention? A. Induced vomiting B. Whole-bowel irrigation C. Administration of activated charcoal D. Administration of fresh frozen plasma

C Administration of activated charcoal Among the most common treatments for poisoning is the administration of activated charcoal. Induced vomiting is not typically indicated, and there is no need for plasma administration. Whole-bowel irrigation may be used as an adjunct therapy later in treatment, but the use of activated charcoal is central to the treatment of poisonings.

The nurse is caring for a patient diagnosed with breast cancer who just underwent an axillary lymph node dissection. What intervention should the nurse use to decrease the lymphedema? A. Keep affected arm flat at the patient's side. B. Apply an elastic bandage on the affected arm. C. Assess blood pressure only on unaffected arm. D. Restrict exercise of the affected arm for 1 week.

C Assess blood pressure only on unaffected arm. Blood pressure readings, venipunctures, and injections should not be done on the affected arm. Elastic bandages should not be used in the early postoperative period because they inhibit collateral lymph drainage. The affected arm should be elevated above the heart, and isometric exercises are recommended and gradually increased starting in the recovery room to reduce fluid volume in the arm.

After an exploratory laparotomy, a patient on a clear liquid diet reports severe gas pains and abdominal distention. Which action by the nurse is most appropriate? a. Return the patient to NPO status. b. Place cool compresses on the abdomen. c. Encourage the patient to ambulate as ordered. d. Administer an as-needed dose of IV morphine sulfate.

C Encourage the patient to ambulate as ordered. Swallowed air and reduced peristalsis after surgery can result in abdominal distention and gas pains. Early ambulation helps restore peristalsis and eliminate flatus and gas pain. Medications used to reduce gas pain include metoclopramide, which stimulates peristalsis. A heating pad can help to alleviate some of the pain and help make the patient more comfortable. There is no need for the patient to return to NPO status. Drinking ginger ale may be helpful.

A 58-yr-old woman is 1-day postoperative after an abdominal hysterectomy. Which intervention should the nurse perform to prevent deep vein thrombosis (DVT)? A. Place the patient in a high Fowler's position. B. Provide pillows to place under the patient's knees. C. Encourage the patient to change positions frequently. D. Teach the patient deep breathing and coughing exercises.

C Encourage the patient to change positions frequently. The patient should be encouraged to change positions frequently and ambulate to prevent venous stasis. The high Fowler's position and pressure under the knees should be avoided in order to prevent DVT. Deep breathing and coughing are therapeutic exercises but do not directly address the risk of DVT.

A 29-yr-old primiparous patient is breastfeeding a 3-wk-old infant. She complains of recent breast tenderness, redness, and fever. Which teaching point should the nurse reinforce when following up on her care? A. Encourage patient to continue breastfeeding her infant. B. Refer patient for a mammogram as quickly as possible. C. Ensure patient adheres to her prescribed antibiotic regimen. D. Teach patient to use warm compresses while explaining the self-limiting nature of illness.

C Ensure patient adheres to her prescribed antibiotic regimen. Mastitis normally requires antibiotic therapy that is closely adhered to. Breastfeeding should be continued if possible, but effective treatment of infection would be the immediate priority. If a palpable mass develops, medical follow-up may be needed to determine if an abscess is present. Mastitis is not necessarily self-limiting, and mammography is not normally indicated.

A patient is admitted to the emergency department after a motor vehicle crash with suspected abdominal trauma. What assessment finding by the nurse is of highest priority? a. Nausea and vomiting b. Hyperactive bowel sounds c. Firmly distended abdomen d. Abrasions on all extremities

C Firmly distended abdomenClinical manifestations of abdominal trauma are guarding and splinting of the abdominal wall; a hard, distended abdomen (indicating possible intraabdominal bleeding); decreased or absent bowel sounds; contusions, abrasions, or bruising over the abdomen; abdominal pain; pain over the scapula; hematemesis or hematuria; and signs of hypovolemic shock (tachycardia and decreased blood pressure).

The condition of a patient who has cirrhosis of the liver has deteriorated. Which diagnostic study would help determine if the patient has developed liver cancer? a. Serum α-fetoprotein level b. Ventilation/perfusion scan c. Hepatic structure ultrasound d. Abdominal girth measurement

C Hepatic structure ultrasound Hepatic structure ultrasonography, CT scan, and MRI are used to screen for and diagnose liver cancer. Serum α-fetoprotein level may be elevated with liver cancer or other liver problems. Ventilation/perfusion scans are used to diagnose pulmonary emboli. Abdominal girth measurement would not differentiate between cirrhosis and liver cancer.

The nurse performs a breast examination on a 68-yr-old female patient. Which clinical manifestation indicates further evaluation for breast cancer is needed? A. Bilateral pendulous breasts B. Right breast is warm, painful to touch C. Irregular, nontender lump with induration D. Palpable lump that is tender and movable

C Irregular, nontender lump with induration Clinical manifestations of breast cancer may include a palpable lump that is hard, irregular, poorly delineated, nonmobile, and nontender. Nipple retraction, peau d'orange, induration, and dimpling of the overlying skin may also be noted. Mastitis presents with breasts that are warm to touch, indurated, and painful. Atrophy of the mammary glands associated with aging may result in pendulous breasts. Manifestations of fibrocystic breast changes include palpable lumps that are round, well delineated, and freely movable. The lump is usually tender and increases in size and tenderness before menstruation.

A nurse teaches the emergency department staff about their roles during a mass casualty incident. Which primary responsibility is expected of all licensed and unlicensed health care staff? A. Notify local, state, and national authorities. B. Assist security personnel to patrol the area. C. Learn the hospital emergency response plan. D. Contact the American Red Cross for assistance.

C Learn the hospital emergency response plan. All health care providers must be prepared for a mass casualty incident. The priority responsibility is to know the agency's emergency response plan.

When doing breast self-examination, the female patient should report which findings to her health care provider? A. Palpable rib margins B. Denser breast tissue C. Left nipple deviation D. Different sized breasts

C Left nipple deviation Unilateral deviation of a nipple may be a clinical indicator of breast cancer or other problem and should be reported to the health care provider. Dense breast tissue, palpable rib margins, and different sized breasts are all normal findings.

A male patient is brought into the emergency department with multiple stab wounds to the legs, one stab wound to the left abdomen, and gang tattoos on both arms. He refused to identify his attacker and then loses consciousness. Police identify him as the assailant in the fatal stabbing of another man. What is the nurse's priority? A. Guard locked access doors. B. Maintain patient safety from revenge. C. Maintain personal and work place safety. D. Attain open patient airway and breathing.

C Maintain personal and work place safety. The nurse's priority is to maintain personal and work place safety. Violence can erupt in the emergency department when treating gang members if the rival gang seeks revenge, or the patient's gang members seek to protect the patient with their presence. Staff members can be victims of that violence, so they should maintain a safe work environment by seeking law enforcement and security assistance in maintaining safety for the staff and the patient. ABCs are the usual priority, but this situation does not show any problem with the patient's airway or breathing.

The nurse is caring for a 52-yr-old woman with breast cancer who is receiving high-dose doxorubicin (Adriamycin). Which assessment is most important for the nurse to make? A. Observe for alopecia. B. Determine visual acuity. C. Monitor cardiac rhythm. D. Assess mouth and throat.

C Monitor cardiac rhythm. Doxorubicin (especially at high doses) may cause cardiotoxicity and heart failure. The nurse should monitor for cardiac dysrhythmias, electrocardiogram changes, and clinical manifestations of heart failure. Other adverse effects of doxorubicin include stomatitis and alopecia, but these effects are not as serious as cardiac problems. Tamoxifen may cause visual changes.

What should the nurse include when teaching about early detection of ovarian cancer? A. Report any pelvic or vaginal bleeding soon. B. Use estrogen with progestin for menopause. C. Obtain annual bimanual pelvic examinations. D. Receive a preventive bilateral oophorectomy.

C Obtain annual bimanual pelvic examinations. Because it is difficult for a patient to detect early clinical indicators of ovarian cancer, the best method of early detection is to have a yearly bimanual pelvic examination to palpate for an ovarian mass. Although pelvic or vaginal bleeding should be reported soon after it occurs, this rarely occurs with ovarian cancer and is not an early symptom. Oral contraceptives may be used or a preventive bilateral oophorectomy may be done to reduce the risk, but they would not be done to detect early ovarian cancer.

A patient with an intestinal obstruction has a nasogastric (NG) tube to suction but complains of nausea and abdominal distention. The nurse irrigates the tube as necessary as ordered, but the irrigating fluid does not return. What should be the priority action by the nurse? a. Notify the physician. b. Auscultate for bowel sounds. c. Reposition the tube and check for placement. d. Remove the tube and replace it with a new one.

C Reposition the tube and check for placement. The tube may be resting against the stomach wall. The first action by the nurse is to reposition the tube and check it again for placement. The physician does not need to be notified unless the nurse cannot restore the tube function. The patient does not have bowel sounds, which is why the NG tube is in place. The NG tube would not be removed and replaced unless it was no longer in the stomach or the obstruction of the tube could not be relieved.

Students are having an end of the semester party, which includes drinking alcohol, having snacks, and swimming. A student was found floating in the pool. Which action by first responders is most important? A. Immobilizing the cervical spine B. Evacuating water from the lungs C. Securing the airway and providing ventilation D. Establishing IV access and infusing warmed fluids

C Securing the airway and providing ventilation Aggressive resuscitation efforts (e.g., airway and ventilation management), especially in the prehospital phase, improve survival of drowning victims. Initial evaluation involves assessment of airway, cervical spine, breathing, and circulation. Treatment of submersion injuries focuses on correcting hypoxia and fluid imbalances, supporting basic physiologic functions, and rewarming when hypothermia is present. Most drowning victims do not aspirate any liquid due to laryngospasm.

A patient with stage I colorectal cancer is scheduled for surgery. Patient teaching for this patient would include an explanation that a. chemotherapy will begin after the patient recovers from the surgery. b. both chemotherapy and radiation can be used as palliative treatments. c. follow-up colonoscopies will be needed to ensure that the cancer does not recur. d. a wound, ostomy, and continence nurse will visit the patient to identify an abdominal site for the ostomy.

C Stage 1 colorectal cancer is treated with surgical removal of the tumor and reanastomosis, and so there is no ostomy. Chemotherapy is not recommended for stage I tumors. Follow-up colonoscopy is recommended because colorectal cancer can recur.

A nurse is teaching a health promotion workshop to a group of women in their 40s and 50s. What information about nipple discharge should the nurse teach to participants? A. Inappropriate lactation necessitates breast biopsy. D. Nipple discharge of any type is considered a precursor to cancer. C. Unexpected nipple discharge of any type warrants medical follow-up. D. Galactorrhea is a normal age-related change and a frequent perimenopausal symptom.

C Unexpected nipple discharge of any type warrants medical follow-up. Although most cases of nipple discharge are not related to malignancy, further medical assessment is indicated. Other testing would be done for inappropriate lactation before a breast biopsy would be necessary. Galactorrhea is not considered a normal age-related change, nor is it a common perimenopausal symptom.

A 24-yr-old patient is at the clinic with symptoms of purulent vaginal discharge, dysuria, and dyspareunia. She is sexually active and has multiple partners. What should the nurse explain as the rationale for Chlamydia screening? A. Chlamydia is frequently comorbid with HIV. B. Chlamydial infections may progress to sepsis. C. Untreated chlamydial infections can lead to infertility. D. Chlamydial infections are treatable only in the early stages of infection.

C Untreated chlamydial infections can lead to infertility. Because of the potential for infertility, routine screening for Chlamydia is recommended for women sexually active younger than age 25 years and annually for those older than 25 years with one or more risk factors for the infection. Chlamydia is not a primary risk for sepsis and is not noted to be strongly correlated with HIV infection. The disease is treatable at all stages of infection.

The history and physical of a 29-yr-old female patient are indicative of human papillomavirus (HPV) infection. What treatment option should be discussed with the patient? A. Gardasil B. Antibiotic therapy C. Wart removal options D. Treatment with antiviral drugs

C Wart removal options Although discussion should focus on the various options for physically removing the symptomatic warts, the removal may or may not decrease infectivity. The HPV vaccine (Gardasil) is ineffective in cases of existing HPV, and neither antiviral nor antibiotic drugs are effective treatments.

The infertile couple is undergoing in vitro fertilization (IVF). The nurse is teaching them about the process. In which order will the steps of IVF occur? (Answer with a letter followed by a comma and a space A. Harvest of mature oocytes B. Implantation of the embryo(s) C. Maturation of ovarian follicles D. In vitro fertilization of the ova with sperm E. Transfer of one or more embryos to the uterus

C, A, D, E, B IVF begins with maturation of the ovarian follicles and then removal of mature oocytes from the follicle via laparoscopy. The oocytes are combined with the sperm in the laboratory and observed for growth. When ready, the embryo(s) is/are transferred to the woman's uterus, and implantation is anticipated.

A nurse has administered corticosteroids to a patient with multiple organ dysfunction syndrome (MODS). In what order should the results of this medication occur in this situation. (Answer with a letter followed by a comma and a space (e.g. A, B, C, D).) a. Renal excretion of potassium b. Increased intravascular volume c. Improved capillary permeability d. Enhanced renal artery perfusion

C, B, D, A

Which clinical manifestations of inflammatory bowel disease does the nurse determine are common to both patients with ulcerative colitis (UC) and Crohn's disease (select all that apply.)? a. Restricted to rectum b. Strictures are common c. Bloody, diarrhea stools d. Cramping abdominal pain e. Lesions penetrate intestine

C, D Clinical manifestations of UC and Crohn's disease include bloody diarrhea, cramping abdominal pain, and nutritional disorders. Intestinal lesions associated with UC are usually restricted to the rectum before moving into the colon. Lesions that penetrate the intestine or cause strictures are characteristic of Crohn's disease.

A chemical explosion occurs at a nearby industrial site. The first responders report that victims are being decontaminated at the scene and approximately 125 workers will need medical evaluation and care. The nurse receiving this report should know that this will first require activation of A. a code blue alert. B. a disaster medical assistance team. C. the local police and fire departments. D. the hospital's emergency response plan.

D

A 60-yr-old woman comes to the clinic reporting unexpected bleeding. What statement should the nurse use regarding diagnosing the cause of bleeding? A. "It is probably the end of menopause." B. "A Pap smear is used to diagnose endometrial cancer." C. "A hysterectomy may be indicated to treat the bleeding." D. "An endometrial biopsy will help determine the cause of bleeding."

D "An endometrial biopsy will help determine the cause of bleeding." With unexpected bleeding in a postmenopausal woman, an endometrial biopsy should be done to exclude or diagnose endometrial cancer. The abnormal bleeding should not be ignored just because she is postmenopausal. A hysterectomy with bilateral salpingo-oophorectomy with lymph node biopsies will be done to treat endometrial cancer if diagnosed. A Pap smear will not diagnose endometrial cancer unless it has spread to the cervix.

The family of a patient newly diagnosed with hepatitis A asks the nurse what they can do to prevent becoming ill. Which response by the nurse is most appropriate? a. "You will need to be tested first; then treatment can be determined." b. "The hepatitis vaccine will provide immunity from this and future exposures." c. "There is nothing you can do since the patient was infectious before admission." d. "An immunoglobulin injection will be given to prevent infection or limit symptoms."

D "An immunoglobulin injection will be given to prevent infection or limit symptoms." Immunoglobulin provides temporary (1-2 months) passive immunity and is effective for preventing hepatitis A if given within 2 weeks after exposure. It may not prevent infection in all persons, but it will at least modify the illness to a subclinical infection. The hepatitis vaccine is only used for preexposure prophylaxis.

While summarizing teaching regarding genital herpes, which patient statement indicates a need for further instruction? A. "No cure is available for my genital herpes." B. "I will utilize my medication when I begin to have symptoms." C. "Genital herpes may be caused by herpes simplex virus type 1 or 2" D. "I am not able to infect a sexual partner unless I have active lesions."

D "I am not able to infect a sexual partner unless I have active lesions." The majority of herpes simplex virus (HSV) transmission occurs during asymptomatic periods. When active lesions are present, the patient is most likely to infect others. There is no cure for HSV, but antiviral medication is prescribed for current infections or suppression of recurrent infections. Early treatment reduces the duration of ulcers and risk of transmission. HSV-1 has been commonly associated with cold sores or fever blisters. HSV-2 has been more associated with genital disease. However, HSV-1 and HSV-2 can cause oral or genital lesions.

When teaching the patient with acute hepatitis C (HCV), which statement demonstrates understanding of the disease process? a. "I will use care when kissing my wife to prevent giving it to her." b. "I will need to take adefovir (Hepsera) to prevent chronic HCV." c. "Now that I have had HCV, I will have immunity and not get it again." d. "I will need to be monitored for chronic HCV and other liver problems."

D "I will need to be monitored for chronic HCV and other liver problems." The majority of patients who acquire HCV usually develop chronic infection, which may lead to cirrhosis or liver cancer. HCV is not transmitted via saliva but by blood exposures such as sharing needles and high-risk sexual activity. The treatment for acute viral hepatitis focuses on resting the body and adequate nutrition for liver regeneration. Adefovir (Hepsera) is taken for severe hepatitis B (HBV) with liver failure. Chronic HCV is treated with pegylated interferon with ribavirin. Immunity with HCV does not occur as it does with HAV and HBV, so the patient may be reinfected with another type of HCV.

The nurse is preparing to administer a dose of bisacodyl to a patient with constipation and the patient asks how it will work. What is the best response by the nurse? a. "It will increase bulk in the stool." b. "It will lubricate the intestinal tract to soften feces." c. "It will increase fluid retention in the intestinal tract." d. "It will increase peristalsis by stimulating nerves in the colon wall."

D "It will increase peristalsis by stimulating nerves in the colon wall." Bisacodyl is a stimulant laxative that aids in producing a bowel movement by irritating the colon wall and stimulating enteric nerves. Fiber and bulk- forming drugs increase bulk in the stool. Water and stool softeners soften feces, and saline and osmotic solutions cause fluid retention in the intestinal tract.

The nurse obtains a history from a 34-yr-old woman diagnosed with a chlamydial infection. Which patient statement indicates additional teaching is required? A. "This infection can be cured by taking antibiotics." B. "It is important to use condoms for all sexual activity." C. "I will avoid sexual contact for 1 week after taking the antibiotics." D. "My sexual partner does not have symptoms and will not need treatment."

D "My sexual partner does not have symptoms and will not need treatment." All sexual partners require treatment. Most men and women with chlamydial infections are either asymptomatic or have minor symptoms. Chlamydial infections are caused by Chlamydia trachomatis, a gram-negative bacterium. Antibiotics will cure this disease. Patients should avoid sexual intercourse for 7 days after completing treatment with antibiotics. Condoms should be used for all sexual contacts.

The nurse teaches a 53-yr-old patient about screening for early detection of breast cancer. Which statement by the patient requires clarification by the nurse? A. "I should plan to have a mammogram every year." B. "I will see a health care provider every year for a breast examination." C. "A breast examination should be done right after my menstrual period." D. "Self-breast examination is a reliable way to detect breast cancer early."

D "Self-breast examination is a reliable way to detect breast cancer early." Screening for the early detection of breast cancer includes yearly mammograms starting at age 45 years. Breast self-examination (BSE) has benefits and limitations and may not be a reliable method for early detection of breast cancer. BSE is optional, but it should be done in premenopausal women right after the menstrual period when the breasts are less lumpy and tender.

The nurse is assisting in the care of several patients in the critical care unit. Which patient is most at risk for developing multiple organ dysfunction syndrome (MODS)? a. A 22-yr-old patient with systemic lupus erythematosus admitted with a pelvic fracture b. A 48-yr-old patient with lung cancer admitted for syndrome of inappropriate antidiuretic hormone and hyponatremia c. A 65-yr-old patient with coronary artery disease, dyslipidemia, and primary hypertension admitted for unstable angina d. A 82-yr-old patient with type 2 diabetes mellitus and chronic kidney disease admitted for peritonitis related to a peritoneal dialysis catheter infection

D A patient with peritonitis is at high risk for developing sepsis. In addition, a patient with diabetes is at high risk for infections and impaired healing. Sepsis and septic shock are the most common causes of MODS. Individuals at greatest risk for developing MODS are older adults and persons with significant tissue injury or preexisting disease. MODS can be initiated by any severe injury or disease process that activates a massive systemic inflammatory response.

A patient with cholelithiasis is being prepared for surgery. Which patient assessment represents a contraindication for a cholecystectomy? a. Low-grade fever of 100°F and dehydration b. Abscess in the right upper quadrant of the abdomen c. Multiple obstructions in the cystic and common bile duct d. Activated partial thromboplastin time (aPTT) of 54 seconds

D Activated partial thromboplastin time (aPTT) of 54 seconds An aPTT of 54 seconds is above normal and indicates insufficient clotting ability. If the patient had surgery, significant bleeding complications postoperatively are very likely. Fluids can be given to eliminate the dehydration. The abscess can be assessed during surgery, and the obstructions in the cystic and common bile duct would be relieved with the cholecystectomy.

Which event discovered during pregnancy would alert the nurse that a cesarean section delivery is indicated? A. Contact with an individual with syphilis 2 weeks ago B. Treatment for gonococcal pharyngitis before conception C. Treatment for Chlamydia trachomatis at her 20th week of gestation D. Active herpes simplex virus type 2 vesicles on her cervix at the time of delivery

D Active herpes simplex virus type 2 vesicles on her cervix at the time of delivery The woman with active herpes simplex virus type 2 at the time of delivery has the greatest risk for the fetus, and the baby will be delivered by cesarean section to prevent infection. Syphilis has an average incubation period of 21 days, so even if the contact was sexual, the syphilis should not infect the baby at birth. The woman treated for gonococcal pharyngitis should have been cured with treatment, but the baby's eyes will be treated at birth to prevent gonorrheal eye infection regardless. Treatment of the pregnant woman with Chlamydia trachomatis prevents transfer of the infection to the fetus.

A mass casualty incident was identified on a nearby freeway. Which patient would likely be designated "red" during triage at the site? A. An individual who is distraught at the violence of the incident B. An individual who has experienced an open arm fracture from falling debris C. An individual who is not expected to survive a crushing head and neck wound D. An individual whose femoral artery has been severed and is bleeding profusely

D An individual whose femoral artery has been severed and is bleeding profusely Red indicates a life-threatening injury requiring immediate intervention, such as severe bleeding. Emotional trauma would not warrant a "red" designation, and a fracture would likely be deemed "yellow," urgent but not life threatening. Those not expected to survive are categorized "blue." "Black" identifies the dead.

The patient with breast cancer has a left mastectomy with axillary node dissection. Ten lymph nodes are resected with three positive for malignant cells. The patient has stage IIB breast cancer. Which nursing intervention would be most effective in planning care? A. Evaluate left arm lymphatic accumulation. B. Maintain joint flexibility and left arm function. C. Teach her about chemotherapy and radiation therapy. D. Assess the patient's response to the diagnosis of breast cancer.

D Assess the patient's response to the diagnosis of breast cancer. Assessment is the first step in planning patient care. Because the nurse is the patient's advocate and this is an extremely stressful time for the patient and family, the nurse should focus on the patient's response to the diagnosis of breast cancer when planning care. The approach for the care of the left arm and teaching the patient about further therapy will be based the initial assessment.

The patient with suspected pancreatic cancer is having many diagnostic studies done. Which one can be used to establish the diagnosis of pancreatic adenocarcinoma and for monitoring the response to treatment? a. Spiral CT scan b. A PET/CT scan c. Abdominal ultrasound d. Cancer-associated antigen 19-9

D Cancer-associated antigen 19-9 The cancer-associated antigen 19-9 (CA 19-9) is the tumor marker used for the diagnosis of pancreatic adenocarcinoma and monitoring the response to treatment. Although a spiral CT scan may be the initial study done and provides information on metastasis and vascular involvement, this test and the positron emission tomography (PET)/CT scan or abdominal ultrasonography does not provide additional information.

The nurse is preparing to administer a scheduled dose of docusate sodium when the patient reports an episode of loose stool and does not want to take the medication. What is the appropriate action by the nurse? a. Write an incident report about this untoward event. b. Attempt to have the family convince the patient to take the ordered dose. c. Withhold the medication at this time and try to administer it later in the day. d. Chart the dose as not given on the medical record and explain in the nursing progress notes.

D Chart the dose as not given on the medical record and explain in the nursing progress notes. Whenever a patient refuses medication, the dose should be charted as not given with an explanation of the reason documented in the nursing progress notes. In this instance, the refusal indicates good judgment by the patient, and the patient should not be encouraged to take it today.

A patient with a history of lung cancer and hepatitis C has developed liver failure and is considering liver transplantation. After a comprehensive evaluation, which finding may be a contraindication for liver transplantation? a. History of hypothyroidism b. Stopped smoking cigarettes c. Well-controlled type 1 diabetes mellitus d. Chest x-ray showed another lung cancer lesion.

D Chest x-ray showed another lung cancer lesion. Contraindications for liver transplant include severe extrahepatic disease, advanced hepatocellular carcinoma or other cancer, ongoing drug or alcohol abuse, and the inability to comprehend or comply with the rigorous posttransplant course.

Because of the risks, a 50-yr-old patient does not want hormone replacement therapy for perimenopausal symptoms. She asks the nurse how to minimize hot flashes and night sweats. What should the nurse recommend first? A. Increase warmth to avoid chills. B. Good nutrition to avoid osteoporosis C. Vitamin B complex and vaginal lubrication D. Decrease heat production and increase heat loss.

D Decrease heat production and increase heat loss. To avoid hot flashes and sweating at night, decrease heat production with a cool environment, limit caffeine and alcohol, and practice relaxation techniques. Heat loss may be facilitated with increased circulation in the room, avoidance of heavy bedding, and wearing loose-fitting clothes. Warmth will facilitate hot flashes. Nutrition, vitamin B complex, and vaginal lubrication will help with other complications of perimenopause but not hot flashes and sweating at night.

The health care provider orders lactulose for a patient with hepatic encephalopathy. Which finding indicates the medication has been effective? a. Relief of constipation b. Relief of abdominal pain c. Decreased liver enzymes d. Decreased ammonia levels

D Decreased ammonia levels Hepatic encephalopathy is a complication of liver disease and is associated with elevated serum ammonia levels. Lactulose traps ammonia in the intestinal tract. Its laxative effect then expels the ammonia from the colon, resulting in decreased serum ammonia levels and correction of hepatic encephalopathy. An additional finding may be an improvement in level of consciousness.

The nurse requests a patient scheduled for colectomy to sign the operative permit as directed in the physician's preoperative orders. The patient states that the physician has not really explained very well what is involved in the surgical procedure. What is the most appropriate action by the nurse? a. Ask family members whether they have discussed the surgical procedure with the physician. b. Have the patient sign the form and state the physician will visit to explain the procedure before surgery. c. Explain the planned surgical procedure as well as possible and have the patient sign the consent form. d. Delay the patient's signature on the consent and notify the physician about the conversation with the patient.

D Delay the patient's signature on the consent and notify the physician about the conversation with the patient. The patient should not be asked to sign a consent form unless the procedure has been explained to the satisfaction of the patient. The nurse should notify the physician, who has the responsibility for obtaining consent.

The nurse is admitting a patient with severe dehydration and frequent watery diarrhea. A 10-day outpatient course of antibiotic therapy for bacterial pneumonia has just been completed. What is the most important for the nurse to take which action? a. Wear a mask to prevent transmission of infection. b. Wipe equipment with ammonia-based disinfectant. c. Instruct visitors to use the alcohol-based hand sanitizer. d. Don gloves and gown before entering the patient's room.

D Don gloves and gown before entering the patient's room. Clostridium difficile is an antibiotic-associated diarrhea transmitted by contact, and the spores are extremely difficult to kill. Patients with suspected or confirmed infection with C. difficile should be placed in a private room, and gloves and gowns should be worn by visitors and health care providers. Alcohol-based hand cleaners and ammonia-based disinfectants are ineffective and do not kill all of the spores. Equipment cannot be shared with other patients, and a disposable stethoscope and individual patient thermometer are kept in the room. Objects should be disinfected with a 10% solution of household bleach.COND

When teaching the patient about the diet for diverticular disease, which foods should the nurse recommend? a. White bread, cheese, and green beans b. Fresh tomatoes, pears, and corn flakes c. Oranges, baked potatoes, and raw carrots d. Dried beans, All Bran (100%) cereal, and raspberries

D Dried beans, All Bran (100%) cereal, and raspberries A high-fiber diet is recommended for diverticular disease. Dried beans, All Bran (100%) cereal, and raspberries all have higher amounts of fiber than white bread, cheese, green beans, fresh tomatoes, pears, corn flakes, oranges, baked potatoes, and raw carrots.

What action should be recommended to a woman recovering from surgical repair of a fistula? A. Douche daily to prevent postoperative infection. B. Remove and cleanse her pessary on a daily basis. C. Resume normal activity to prevent adhesion formation. D. Ensure that she does not place stress on the repaired area.

D Ensure that she does not place stress on the repaired area. After surgical repair of a fistula, the patient should avoid placing stress on the repaired region. Normal activity is not commonly resumed until significant healing has occurred. Douching is contraindicated, and pessaries are used to treat prolapses, not fistulas.

A 30-yr-old woman reports the recent appearance of itchy lesions on her vulva, some of which have recently burst. Which STI should the nurse suspect first? A. HIV B. Gonorrhea C. Chlamydia D. Genital herpes

D Genital herpes A primary episode of genital herpes is often marked by multiple small, vesicular lesions on the genitals. This symptomatology is not commonly associated with HIV, gonorrhea, or Chlamydia.

The nurse is caring for a patient admitted with a suspected bowel obstruction. The nurse auscultating the abdomen listens for which type of bowel sounds that are consistent with the patient's clinical picture? a. Low-pitched and rumbling above the area of obstruction b. High-pitched and hypoactive below the area of obstruction c. Low-pitched and hyperactive below the area of obstruction d. High-pitched and hyperactive above the area of obstruction

D High-pitched and hyperactive above the area of obstruction Early in intestinal obstruction, the patient's bowel sounds are hyperactive and high pitched, sometimes referred to as "tinkling," above the level of the obstruction. This occurs because peristaltic action increases to "push past" the area of obstruction. As the obstruction becomes complete, bowel sounds decrease and finally become absent.

A patient with hepatitis B surface antigen (HBsAg) present in the serum is being discharged with pain medication after knee surgery. Which medication order should the nurse question? a. Tramadol b. Hydromorphone (Dilaudid) c. Oxycodone with aspirin (Percodan) d. Hydrocodone with acetaminophen

D Hydrocodone with acetaminophen The analgesic with acetaminophen should be questioned because this patient is a chronic carrier of hepatitis B and is likely to have impaired liver function. Acetaminophen is not suitable for this patient because it is converted to a toxic metabolite in the liver after absorption, increasing the risk of hepatocellular damage.

When planning care for a patient with cirrhosis, the nurse will give highest priority to which nursing diagnosis? a. Impaired skin integrity related to edema, ascites, and pruritus b. Imbalanced nutrition: less than body requirements related to anorexia c. Excess fluid volume related to portal hypertension and hyperaldosteronism d. Ineffective breathing pattern related to pressure on diaphragm and reduced lung volume

D Ineffective breathing pattern related to pressure on diaphragm and reduced lung volume Although all of these nursing diagnoses are appropriate and important in the care of a patient with cirrhosis, airway and breathing are always the highest priorities.

The nurse is administering a cathartic agent to a patient with renal insufficiency. Which order will the nurse question? a. Bisacodyl b. Lubiprostone c. Cascara sagrada d. Magnesium hydroxide

D Magnesium hydroxide Milk of Magnesia may cause hypermagnesemia in patients with renal insufficiency. The nurse should question this order with the health care provider. Bisacodyl, lubiprostone, and cascara sagrada are safe to use in patients with renal insufficiency as long as the patient is not currently dehydrated.

A 51-yr-old woman has recently had a unilateral, right total mastectomy and axillary node dissection for the treatment of breast cancer. What nursing intervention should the nurse include in the patient's care? A. Immobilize the patient's right arm until postoperative day 3. B. Maintain the patient's right arm in a dependent position when at rest. C. Administer diuretics prophylactically for the prevention of lymphedema. D. Promote gradually increasing mobility as soon as possible following surgery.

D Promote gradually increasing mobility as soon as possible following surgery. Mobility should be encouraged beginning in the postanesthesia care unit (PACU) and increased gradually throughout the patient's recovery. Immobilization is counterproductive to recovery, and the limb should not be in a dependent position. Diuretics are not used to prevent lymphedema but may be used in active treatment of the problem.

After coronary artery bypass graft surgery a patient has postoperative bleeding that requires returning to surgery for repair. During surgery, the patient has a myocardial infarction (MI). After restoring the patient's body temperature to normal, which patient parameter is the most important for planning nursing care? a. Cardiac index (CI) of 5 L/min/m2 b. Central venous pressure of 8 mm Hg c. Mean arterial pressure (MAP) of 86 mm Hg d. Pulmonary artery pressure (PAP) of 28/14 mm Hg

D Pulmonary hypertension as indicated by an elevated PAP indicates impaired forward flow of blood because of left ventricular dysfunction or hypoxemia. Both can be a result of the MI. The CI, CVP, and MAP readings are normal.

A 55-yr-old woman diagnosed with endometrial cancer is receiving brachytherapy. The nurse is most concerned if what is observed? A. Foul-smelling vaginal discharge B. 5 to 8 liquid stools over a period of 24 hours C. Use of a bedpan instead of a bedside commode or toilet. D. Request for a nursing assistant to stay in the room for company.

D Request for a nursing assistant to stay in the room for company. Brachytherapy is internal radiation applied directly to the tumor. Health care providers should limit close contact with the patient to less than 30 minutes per day. Internal radiation causes the destruction of cells and results in a foul-smelling vaginal discharge. Internal radiation may cause systemic reactions such as nausea, vomiting, diarrhea, and malaise. The patient receiving brachytherapy is placed in a lead-lined private room and on absolute bed rest.

A woman with endometriosis is seeking a cure. After identifying childbearing is no longer desired, the nurse should introduce which potential treatment? A. Danazol B. Leuprolide (Lupron) C. Nonsteroidal antiinflammatory drugs D. Surgical removal of endometrial implants

D Surgical removal of endometrial implants The only cure for endometriosis is the surgical removal of all endometrial implants, which may include the uterus, fallopian tubes, and ovaries. Leuprolide is a gonadotropin-releasing hormone agonist that causes amenorrhea with menopausal side effects. Danazol is a synthetic androgen that inhibits the anterior pituitary. Nonsteroidal antiinflammatory drugs relieve pain but do not affect the problem of endometriosis.

The nurse is preparing to administer a daily dose of docusate sodium to a patient that will continue taking it after discharge. What information should the nurse provide to the patient to optimize the outcome of the medication? a. Take a dose of mineral oil at the same time. b. Add extra salt to food on at least one meal tray. c. Ensure a dietary intake of 10 g of fiber each day. d. Take each dose with a full glass of water or other liquid.

D Take each dose with a full glass of water or other liquid. Docusate lowers the surface tension of stool, permitting water and fats to penetrate and soften the stool for easier passage. The patient should take the dose with a full glass of water and should increase overall fluid intake, if able, to enhance effectiveness of the medication. Dietary fiber intake should be a minimum of 20 g daily to prevent constipation. Mineral oil and extra salt are not recommended.

A massive gastrointestinal bleed has resulted in hypovolemic shock in an older patient. What is a priority nursing diagnosis? a. acute pain b. impaired skin integrity c. decreased CO d. ineffective tissue perfusion

D The many deleterious effects of shock are all related to inadequate perfusion and oxygenation of every body system. This nursing diagnosis supersedes the other diagnoses.

What laboratory finding is consistent with a medical diagnosis of cardiogenic shock? a. Decreased liver enzymes b. Increased white blood cells c. Decreased red blood cells, hemoglobin, and hematocrit d. Increased blood urea nitrogen (BUN) and serum creatinine (Cr) levels

D The renal hypoperfusion that accompanies cardiogenic shock results in increased BUN and creatinine levels. Impaired perfusion of the liver results in increased liver enzymes, but white blood cell levels do not typically increase in cardiogenic shock. Red blood cell indices are typically normal because of relative hypovolemia.

Facilitating the nutritional status of a patient early in the starvation cycle is important. In what order do the following events occur in starvation? (Answer with a letter followed by a comma and a space (e.g. A, B, C, D).) a. Body fat is mobilized to supply energy.b. Skeletal proteins are converted to glucose for energy.c. Body or visceral proteins are broken down to supply glucose.d. Glycogen stores in liver and muscle are broken down to glucose.

DBAC

A patient is jaundiced and her stools are clay colored (gray). This is most likely related to a. decreased bile flow into the intestine b. increased production of urobilirubin c. increased bile and bilirubin in the blood d. increased production of cholecystokinin

a

A patient with hepatitis A is in the acute phase. The nurse plans care for the patient based on the knowledge that a. pruritus is a common problem with jaundice in this phase b. the patient is most likely to transmit the disease during this phase c. GI symptoms are not as severe in hepatitis A as they are in hepatitis B d. extrahepatic manifestations of glomerulonephritis and polyarteritis are common in this phase

a

Always assess the patient with an opthalmic problem for a. visual acuity b. pupillary reactions c. intraocular pressure d. confrontation visual fields

a

An older man arrives in triage disoriented and dyspneic. His skin is hot and dry. His wife states that he was fine earlier today. The nurse's next priority would be to a. assess his vital signs. b. obtain a brief medical history from his wife. c. start supplemental O2 and have the ED physician see him. d. determine the kind of insurance he has before treating him.

a

In planning care for a patient with metastatic liver cancer, the nurse should include interventions that a. focus primarily on symptomatic and comfort measures b. reassure the patient that chemotherapy offers a good prognosis c. promote the patient's confidence that surgical excision of the tumor will be successful d. provide information necessary for the patient to make decisions regarding liver transplantation

a

Inflammation and infection of the eye a. are caused by irritants and microorganism b. have a higher incidence in sexually active patients c. are chronic problems that result in loss of vision d. are frequently treated with cold compresses and antibiotics

a

Nursing responsibilities related to the patient receiving brachytherapy for endometrial cancer include a. maintaining absolute bed rest b. keeping the patient in high Fowler's position c. allowing visitors if they stay 3 ft (1 m) from the bed d. limiting direct nurse-to-patient contact to 30 minutes per shift

a

The first nursing intervention for the patient who has been sexually assaulted is to a. treat urgent medical problems b. contact support person for the patient c. provide supplies for the patient to cleanse self d. document bruises and lacerations of the perineum and the cervix

a

The nurse determines a patient undergoing ileostomy surgery understands the procedure when the patient states a. "I should only have to change the pouch every 4-7 days" b. "The drainage in the pouch will look like my normal stools" c. "I may not need to wear a drainage pouch if I irrigate it daily" d. "Limiting my fluid intake should decrease the amount of output"

a

The nurse determines that the goals of dietary teaching have been met when the patient with celiac disease selects from the menu a. scrambled eggs and sausage. b. buckwheat pancakes with syrup. c. oatmeal, skim milk, and orange juice. d. yogurt, strawberries, and rye toast with butter.

a

The nurse is obtaining a subjective data assessment from a woman reported as sexual contact of a man with chlamydia infection. The nurse understands that symptoms of chlamydial infection in a women a. are frequently absent b. are similar to those of genital herpes c. include a macular palmar rash in the later stages d. may involve chancres inside the vagina that are not visible

a

The patient who has a conductive hearing loss a. heads better in a noisy environment b. heads sound but does not understand speech c. often speaks loudly because his or her own voice seems low d. experiences clearer sound with a hearing aid if the loss is less than 30 dB

a

The percentage of daily calories for a healthy person consists of a. 50% carbohydrates, 25% protein, 25% fat, and <10% of fat from saturated fatty acids b. 65% carbohydrates, 25% protein, 25% fat, and >10% of fat from saturated fatty acids c. 50% carbohydrates, 40% protein, 10% fat, and <10% fat from saturated fatty acids d. 40% carbohydrates, 30% protein, 30% fat, and >10% of fat from saturated fatty acids

a

To prevent capsular formation after breast reconstruction with implants, teach the patient to a. gently massage the area around the implant b. bind the breasts tightly with elastic bandages c. exercise the arm on the affected side to promote drainage d. avoid strenuous exercise until the implant has healed

a

When discussing risk factors for breast cancer with a group of women, you emphasize that the greatest known risk factor for breast cancer is a. being a woman over age 60 b. experiencing menstruation for 30 years or more c. using hormone therapy for 5 years for menopausal symptoms d. having a paternal grandmother with postmenopausal breast cancer

a

Which instructions would the nurse include in a teaching plan for a patient with mild GERD? a. "the best time to take an antacid is 1 to 3 hours after meals" b. "a glass of warm milk at bedtime will decrease your discomfort at night" c. "do not chew gum; the excess saliva will cause you to secrete more acid" d. "limit your intake of foods high in protein because they take longer to digest"

a

Which method is best to use when confirming inital placement of a blindly inserted small bore NG feeding tube? a. x-ray b. air insufflation c. observing patient for coughing d. pH measurement of gastric aspirate

a

Which patient behaviors would the nurse promote for healthy eyes and ears (SATA) a. wearing protective sunglasses when bicycling b. supplemental intake of vitamin B and magnesium c. playing amplified music at 75 % of maximum volume d. Patient notifying the health care provider of tinnitus while on antibiotics e. A women avoiding pregnancy for 4 weeks after receiving MMR immunization

a

The appropriate collaborative therapy for the patient with acute diarrhea caused by a viral infection is to a. increase fluid intake. b. administer an antibiotic. c. administer antimotility drugs. d. quarantine the patient to prevent spread of the virus.

a Acute diarrhea resulting from infectious causes (e.g., virus) is usually self-limiting. The major concerns are transmission prevention, fluid and electrolyte replacement, and resolution of the diarrhea. Antidiarrheal agents are contraindicated in the treatment of infectious diarrhea because they potentially prolong exposure to the infectious organism. Antibiotics are rarely used to treat acute diarrhea. To prevent transmission of diarrhea caused by a virus, hands should be washed before and after contact with the patient and when body fluids of any kind are handled. Vomitus and stool should be flushed down the toilet, and contaminated clothing should be washed immediately with soap and hot water.

Assessment findings suggestive of peritonitis include a. rebound abdominal pain. b. a soft, distended abdomen. c. dull, continuous abdominal pain. d. observing that the patient is restless.

a With peritoneal irritation, the abdomen is hard, like a board, and the patient has severe abdominal pain that is worse with any sudden movement. The patient lies very still. Palpating the abdomen and releasing the hands suddenly causes sudden movement within the abdomen and severe pain. This is called rebound tenderness.

A patient with breast cancer has a lumpectomy with sentinel lymph node dissection that is positive for cancer. You explain that, of the other tests done to determine the risk for cancer recurrence or spread, the results that support the more favorable prognosis are (select all that apply) a. well-differentiated tumor b. estrogen receptor-positive tumor c. overexertion of HER-2 cell marker d. involvement of two to four axillary nodes e. aneuploidy status from cell proliferation studies

a, b

The nurse performs a detailed assessment of the abdomen of a patient with a possible bowel obstruction, knowing that manifestations of an obstruction in the large intestine are (select all that apply) a. persistent abdominal pain. b. marked abdominal distention. c. diarrhea that is loose or liquid. d. colicky, severe, intermittent pain. e. profuse vomiting that relieves abdominal pain.

a, b With lower intestinal obstructions, abdominal distention is markedly increased and pain is persistent. Onset of a large intestine obstruction is gradual, vomiting is rare, and there is usually absolute constipation, not diarrhea.

In assessing patients with STIs, the nurse needs to know that many STIs can be assymptomatic. Which STIs can be asymptomatic (select all that apply)? a. syphillis b. gonorrhea c. genital warts d. genital herpes e. chlamydia infection

a, b, c, d, e

When a 35-year-old female patient is admitted to the emergency department with acute abdominal pain, which possible diagnosis should you consider that may be the cause of her pain (select all that apply)? a. Gastroenteritis b. Ectopic pregnancy c. Gastrointestinal bleeding d. Irritable bowel syndrome e. Inflammatory bowel disease

a, b, c, d, eR All these conditions could cause acute abdominal pain.

Postoperative goals in caring for the patient who has undergone an abdominal hysterectomy include (select all that apply) a. monitoring urine output b. changing position frequently c. restricting all food for 24 hours d. observing perineal pad for bleeding e. encouraging leg exercises to promote circulation

a, b, e

Appropriate treatment modalities for the management of cardiogenic shock include (select all that apply) a. dobutamine to increase myocardial contractility. b. vasopressors to increase systemic vascular resistance. c. circulatory assist devices such as an intraaortic balloon pump. d. corticosteroids to stabilize the cell wall in the infarcted myocardium. e. Trendelenburg positioning to facilitate venous return and increase preload.

a, c

What are effective interventions to decrease absorption or increase elimination of an ingested poison? SATA a. hemodialysis b. milk dilution c. eye irrigation d. gastric lavage e. activated charcoal

a, d, e

Nursing management of the patient with acute pancreatitis includes (SATA) a. checking for signs of hypocalcemia b. providing a diet low in carbs c. giving insulin based on a sliding scale d. monitoring for infection, particularly respiratory tract infection

a, e

Which criteria must be met for a diagnosis of metabolic syndrome (SATA) a. hypertension b. elevated triglycerides c. elevated plasma glucose d. increased waist circumference e. decreased low density lipoproteins

abcd

Normal physical assessment finding of the GI system are (SATA) a. nonpalpable liver and spleen b. borborygmi in upper right quadrant c. tympany on percussion of the abdomen d. liver edge 2 to 4 cm below the costal margin e. finding of a firm, nodular edge on the rectal examination

ac

Age related changes in the auditory system commonly include (SATA) a. drier cerumen b. tinnitus in both ears c. auditory nerve degeneration d. atrophy of the tympanic membrane e. greater ability to hear high pitched sounds

acd

Health risks associated with obesity include (SATA) a. colorectal cancer b. rheumatoid arthritis c. polycystic ovary syndrome d. nonalcoholic steatohepatitis e. systemic lupus erythematosus

acd

Before injecting fluorescein for angiography, it is important for the nurse to (SATA) a. obtain an emesis basin b. ask if the patient is fatigued c. administer a topical anesthetic d. inform patient that skin may turn yellow e. assess for allergies to iodine-based contrast media

ad

What should be included in the postoperative teaching of the patient who has undergone cataract surgery (SATA) a. eye discomfort is often relieved with mild analgesic b. a decline in visual acuity is common for the first week c. stay on bed rest and limit activity for the first few days d. notify surgeon if an increase in redness or drainage occurs e. nighttime eye shielding and activity restrictions are essential to prevent eyestrain

ad

A 78-yr-old man has confusion and temperature of 104 degrees F. He is a diabetic with purulent drainage from his right heel. After an infusion of 3 L of NS solution, his assessment findings are BP 84/40, HR 110, RR 42 and shallow, CO 8 L/min, and PAWP 4 mmHg. This patient's symptoms are most likely indicative of a. sepsis b. septic shock c. MODS d. systemic inflammatory response syndrome

b

A patient has a core temperature of 90 degrees F. The most appropriate rewarming technique would be a. passive rewarming with warm blankets b. active internal rewarming using warmed IV fluids c. passive rewarming using air-filled warming blankets d. active external rewarming by submersing in a warm bath

b

A patient has an elevated blood level of indirect (unconjugated) bilirubin. One caused of this is that a. the gallbladder is unable to contract to release stored bile b. bilirubin is not being conjugated and excreted into the bile by the liver c. the kupffer cells in the liver are unable to remove bilirubin from the blood d. there is an obstruction in the biliary tract preventing flow of bile into the small intestine

b

A patient has e=been told that she has elevated liver enzymes caused by nonalcoholic fatty liver disease (NAFLD). The nursing teaching plan should include a. having genetic testing done b. recommending a heart-healthy diet c. the necessity to reduce weight rapidly d. avoiding alcohol until liver enzymes return to normal

b

A patient is receiving peripheral parenteral nutrition. The parenteral nutrition solution is complpeted before the new solution arrives on the unit. The nurse gives a. 20% intralipids b. 5% dextrose solution c. 0.45% normal saline solution d. 5% lactated Ringer's solution

b

A patient with acute hepatitis B is being discharged in 2 days. The discharge teaching plan should include instructions to a. avoid alcohol for the first 3 weeks b. use a condom during sexual intercourse c. have family members get an injection of immunoglobulin d. follow a low-protein, moderate-carb, moderate-fat diet

b

An 80 year old man states that, although he adds a lot of salt to his food, it still does not have much taste. The nurse's response is based on knowledge that the older adult a. should not experience changes in taste b. has a loss of taste buds, especially for sweet and salt c. has some loss of taste but no difficulty chewing food d. loses the sense of taste because the ability to smell is decreased

b

As gastric contents move into the small intestine, the bowel is normally protected from the acidity of gastric contents by the a. inhibition of secretin release b. release of bicarbonate by the pancreas c. release of pancreatic digestive enzymes d. release of gastrin by the duodenal mucosa

b

During an examination of the abdomen the nurse should a.position the patient in the supine position with the bed flat and knees straight b. listen for bowel sounds in the epigastrium and all four quardrants for 2 mins c. describe bowel sounds as absent if no sound is heard in all quadrants after 2 minutes d. use the following order of techniques: inspection, palpitation, percussion, auscultation

b

In a patient who has a hemorrhage in the posterior cavity of the eye, the nurse knows that blood is accumulating a. in aqueous humor b. between the lens and retina c. between the cornea and lens d. in the space between the iris and lens

b

In caring for a patient with pelvic inflammatory disease, the nurse should place her in semi-Fowler's position. The rationale for this is to a. relieve severe pain b. promote drainage to prevent abscesses c. improve circulation and promote healing d. prevent complication of bowel obstruction

b

In contrast to diverticulitis, the patient with diverticulosis a. has rectal bleeding b. often has no symptoms c. has localized cramping pain d. frequently develops peritonitis

b

In preparing a patient for a colonoscopy, the nurse explains that a. signed permit is not necessary b. sedation will be used during the procedure c. one cleansing enema is necessary for preparation d. light meals should be eaten for 3 days before the procedure

b

In telling a patient with infertility what she and her partner can expect, the nurse explains that a. ovulatory studies can help determine tube patency b. a hysterosalpingogram is a common diagnostic study c. the cause will remain unexplained for 40% of couples d. if postcoital studies are normal, infection tests will be done

b

M.J. calls the clinic and tells the nurse that her 85 year old mother has been nauseated all day and has vomited twice. Before the nurse hangs up and calls the HCP, should tell M.J. to a. administer antiemetic drugs and observe skin turgor b. give her mother sips of water and elevate the head of her bed to prevent aspiration c. offer her other a high protein liquid supplement to drink to maintain her nutritional needs d. offer her mother large quanities of gatorade to decrease the risk of sodium depletion

b

Provide emotional support to a patient with an STI by a. offering information on how safer sexual practices can prevent STIs b. showing concern when listening to the patient who expresses negative feelings c. reassuring the patient that the disease is highly curable with appropriate treatment d. helping the patient who received an STI from his or her sexual partner in forgiving the partner

b

Several patients are seen at an urgent care center with symptoms of nausea, vomiting, and diarrhea that began 2 hours ago while attending a large family reunion potluck dinner. You question the patients specifically about foods they ingested containing a. beef b. meat and milk c.poultry and eggs d. home preserved vegetables

b

Teaching in relation to home management after a laparoscopic cholecystectomy should include a. keeping the bandages on the puncture site for 48 hours b. reporting any bile-colored drainage or pus from any incision c. using over-the-counter antiemetics if cause and vomiting occur d. emptying and measuring the contents of the bile bag from the T tube every day

b

The individual with the lowest risk for sexually transmitted pelvic inflammatory disease in the woman who uses a. oral contraceptives b. barrier methods of contraception c. an intrauterine device for contraception d. norplant implant or injectable depo-provedra for contraception

b

The patient with advanced cirrhosis asks why his abdomen is so swollen. The nurse's response is based on the knowledge that a. a lack of clotting factors promotes the collection of blood in the abdominal cavity b. portal hypertension and hypoalbuminemia cause a fluid shift into the peritoneal space c. decreased peristalsis in the GI tract contributes to gas formation and distention of the bowel d. bile salts in the blood irritate the peritoneal membranes, causing edema and pocketing of fluid

b

The patient's thick, white, and curd-like vaginal discharge and vulvar pruritus are most consistent with a. trichomoniasis b. monilial vaginitis c. bacterial vaginosis d. chlamydial cervicitis

b

presbyopia occurs in older individuals because a. the eyeball elongates b. the lens becomes inflexible c. the corneal curvature becomes irregular d. light rays are focusing in front of the retina

b

Place in order the substrates the body uses for energy during starvation, beginning with 1 for the first component and ending with for the last component a. skeletal protein b. glycogen c. visceral protein d. fat stores

b a d c

A primary HSV infection differs from recurrent HSV episodes in that (select all that apply) a. only primary infections are sexually transmitted b. symptoms are less severe during recurrent episodes c. transmission of the virus to a fetus is less likely during pregnancy d. systemic manifestations such as fever and myalgia are more common in primary infection e. lesions from recurrent HSV are more likely to transmit the virus than lesions from a primary HSV

b, d

Instruct the patient who is newly fitted with bilateral hearing aids to SATA a. replace the batteries monthly b. clean the ear molds weekly or as needed c. clean ears with cotton-tipped applicators daily d. disconnect or remove the batteries when not in use e. Initially restrict usage to quiet listening in the home

bde

Which strategies would best assist the nurse in communicating with a patient who has a hearing loss (SATA) a. overnunciate speech b. speak normally and slowly c. exaggerate facial expression d. raise the voice to a higher pitch e. write out names or difficult words

be

A patient who has undergone an esophargectomy for esophageal cancer develops increasing pain, fever, and dyspnea when a full liquid diet is started postoperatively. The nurse recognizes that these symptoms are most indicative of a. an intolerance to the feedings b. extension of the tumor into the aorta c. leakage of fluids into the mediastinum d. esophageal perforation with fistula formation into the lung

c

A patient with pancreatic cancer is admitted to the hospital for evaluation of possible treatment options. The patient asks the nurse to explain the Whipple procedure that the surgeon has described. The explanation includes the information that a Whipple procedure includes a. creating a bypass around the obstruction caused by the tumor by joining the gallbladder to the jejunum b. resection of the entire pancreas and the distal portion of the stomach, with anastomosis of the common bile duct and the stomach into the duodenum c. removal of part of the pancreas, part of the stomach, the duodenum, and the gallbladder, with joining of the pancreatic duct, the common bile duct, and the stomach into the jejunum d. radical removal of the pancreas, the duodenum, and the spleen, and attachment of the stomach to the jejunum, which requires oral supplementation of pancreatic digestive enzymes and insulin replacement therapy

c

An older woman arrives in the ED complaining of severe pain in her right shoulder. The nurse notes that her clothes are soiled with urine and feces. She tells the nurse that she lives with her son and that she "fell". She is tearful and asks you if she can be admitted. What possibility should the nurse consider? a. dementia b. possible cancer c. family violence d. orthostatic hypotension

c

During an assessment of hearing, the nurse would expect that a normal finding would be a. absent cone of light b. bluish purple tympanic membrane c. midline tone heard equally in both ears d. fluid level at hair line in the tympanum

c

Preoperatively, to meet the psychologic needs of a woman scheduled for a modified radical mastectomy, you would a. discuss the limitations of breast reconstruction b. include her significant other in all conversations c. promote an environment for expression of feelings d. explain the importance of regular follow-u screening

c

Question patients using eye drops to treat their glaucoma about a. use of corrective lenses b. their usual sleep pattern c. a history of heart or lung disease d. sensitivity to opioids or depressants

c

The nurse should advise the woman recovering from surgical treatment of an ectopic pregnancy that a. she has an increased risk for salpingitis b. bed rest must be maintained for 12 hours to assist in healing c. having one ectopic pregnancy increases her risk for another d. intrauterine devices and infertility treatments should be avoided

c

The nurse teaching young adults about behaviors that put them at risk for oral cancer includes a. discouraging use of chewing gum b. avoiding use of perfumed lip gloss c. avoiding use of smokeless tobacco d. discouraging drinking of carbonated beverages

c

The pernicious anemia that may accompany gastritis is due to a. chronic autoimmune destruction of cobalamin stores in the body b. progressive gastric atrophy from chronic breakage in the mucosal barrier and blood loss c. a lack of intrinsic factor normally produced by acid-secreting cells of the gastric mucosa d. hyperchlorhydria from an increase in acid-secreting parietal cells and degradation of RBCs

c

To prevent or decrease age-related changes that occur after menopause in a patient who chooses not to take hormone therapy, the most important self-care measure to teach is a. maintaining usual sexual activity b. increasing the intake of dairy products c. performing regular, aerobic, weight-bearing exercise d. taking vitamin E and B-complex vitamin supplements

c

What should be included in the nursing plan for a patient who needs to administer antibiotic therapy? a. cool the drops so that they decrease swelling in the canal b. Avoid placing a cotton wick to assist in administering the drops c. Be careful to avoid touching the tip of the dropper bottle to the car d. Keep the head tilted 5 to 7 minutes after administration of the drops

c

When the nurse is assessing the health perception-health maintenance pattern as related to GI function, an appropriate question to ask is a. "what is your usual bowel elimination pattern?" b. "what percentage of your income is spent on food?" c. "have you traveled to a foreign country in the last year?" d. "Do you have diarrhea when you are under a lot of stress?"

c

Which statement best describes the etiology of obesity? a. obesity primarily results from a genetic predisposition b. psychosocial factors can override the effects of genetics in the etiology of obesity c. obesity is the result of complex interactions between genetic and environmental factors. d. genetic factors are more important than environmental factors in the etiology of obesity

c

In planning care for the patient with Crohn's disease, the nurse recognizes that a major difference between ulcerative colitis and Crohn's disease is that Crohn's disease a. frequently results in toxic megacolon. b. causes fewer nutritional deficiencies than ulcerative colitis. c. often recurs after surgery, whereas ulcerative colitis is curable with a colectomy. d. is manifested by rectal bleeding and anemia more frequently than is ulcerative colitis.

c Ulcerative colitis affects only the colon and rectum; it can cause megacolon and rectal bleeding, but not nutrient malabsorption. Surgical removal of the colon and rectum cures it. Crohn's disease usually involves the ileum, where bile salts and vitamin cobalamin are absorbed. After surgical treatment, disease recurrence at the site is common.

You are a community health nurse planning a program on breast cancer screening guidelines for women in the neighborhood. To best promote the participants' learning and adherence, you would include (select all that apply) a. a short audiotape on the BSE procedure b. a packet of articles from the medical literature c. written guidelines for mammography and CBE d. a discussion of the value of early breast cancer detectione. community resources where they can obtain an ultrasound and MRI

c, d

The teaching plan for the patient being discharged after an acute episode of upper GI bleeding includes information concerning the importance of (SATA) a. limiting alcohol intake to one serving per day b. only taking aspirin with milk or bread products c. avoiding taking aspirin and drugs containing aspirin d. only taking drugs prescribed by the health care provider e. taking all drugs 1 hour before mealtime to prevent further bleeding

cd

A complete nutritional assessment including anthropometric measurements is important for the patient who a. has a BMI of 25.5 kg/m2. b. complains of frequent nocturia. c. reports a 5-year history of constipation. d. reports an unintentional weight loss of 10 lb in 2 months.

d

A modified radical mastectomy has been scheduled for your patient with breast cancer. Postoperatively, to restore arm function on the affected side, you would a. apply heating pads or blankets to increase circulation b. place daily ice packs to minimize the risk of lymphedema c. teach passive exercises with the affected arm in a dependent position d. emphasize regular exercises for the affected shoulder to increase range of motion

d

A nursing intervention that is most appropriate to decrease preoperative edema and pain after an inguinal herniorrhaphy is a. applying a truss to the hernia site b. allowing the patient to stand to void c. supporting the incision during coughing d. applying a scrotal support with an ice bag

d

A patient has a spinal cord injury at T4. Vital signs include falling blood pressure with bradycardia. The nurse recognizes that the patient is experiencing a. a relative hypervolemia b. an absolute hypovolemia c. neurogenic shock from low blood flow d. neurogenic shock from massive vasodilation

d

A patient is admitted to the hospital with a diagnosis of diarrhea with dehydration. The nurse recognizes that increased peristalsis resulting in diarrhea can be related to a. sympathetic inhibition b. mixing and propulsion c. sympathetic stimulation d. parasympathetic stimulation

d

A patient with anorexia nervosa shows signs of malnutrition. During initial refeeding, the nurse carefully assesses the patient for a. hyperkalemia b. hypoglycemia c. hypercalcemia d. hypophosphatemia

d

A patient with extreme obesity has undergone Roux-en-Y gastric bypass surgery. In planning postoperative care, the nurse anticipates that the patient a. may have severe diarrhea early in the postoperative period b. will not be allowed to ambulate to 1 to 2 days postoperative c. will require nasogastric suction until the drainage is pale yellow d. may have only liquids orally, and in very limited amounts during the early postoperative period

d

An appropriate question to ask the patient with painful menstruation to differentiate primary from secondary dysmenorrhea is a. "does your pain become worse with activity or overexertion?" b. "have you had a recent personal crisis or change in your lifestyle?" c. "is your pain relieved by non steroidal anti-inflammatory medications?" d. "when in your menstrual history did the pain with your period begin?"

d

An optimal teaching plan for an outpatient with stomach cancer receiving radiation therapy should include information about a. cancer support groups, alopecia, and stomatitis b. nutrition supplements, ostomy care, and support groups c. prosthetic devices, wound, and skin care, and grief counseling d. wound and skin care, nutrition drugs, and community resources

d

Explain to the patient with gonorrhea that treatment will include both ceftriaxone and azithromycin because a.azithromycin helps prevent recurrent infections b. some patients do not respond to oral drugs alone c. coverage with more than one antibiotic will prevent reinfection d. the increasing rates of drug resistance require the use of at least two drugs

d

In teaching a patient who wants to perform BSE, you inform her that the technique involves both the palpation of the breast tissue and a. palpation of the cervical lymph nodes b. hard squeezing of the breast tissue c. a mammogram to evaluate breast tissue d. inspection of the breasts for any changes

d

Increased intraocular pressure may occur as a result of a. edema of the corneal stroma b. dilation of the retinal arterioles c. blockage of the lactrimal canals and ducts d. increased production of aqueous humor by the ciliary process

d

Patients with permanent visual impairment a.feel most comfortable with other visually impaired people b. may fed threatened when others make eye contact during a conversation c. usually need others to speak louder so that they can communicate appropriately d. may experience the same grieving process that is associated with other losses

d

The best nutritional therapy plan for a person who is obese is a. the zone diet b. the atkins diet c. sugar busters d. foods from the basic food groups

d

The most accurate assessment parameters for the nurse to use to determine adequate tissue perfusion in the patient with MODS are a. blood pressure, pulse, and respirations. b. breath sounds, blood pressure, and body temperature. c. pulse pressure, level of consciousness, and pupillary response. d. level of consciousness, urine output, and skin color and temperature.

d

The most important intervention for the patient with epidemic keratoconjunctivitis is a. cleansing the affect area with baby shampoo b. monitoring spread of infection to the opposing eye c. regular instillation of artificial tears to the affected eye d. teaching the patient and family members good hygiene techniques

d

The nurse explains to the patient with Vincent's infection that treatment will include a. tetanus vaccination b. viscous lidocaine rinses c. amphotericin B suspension d. topical application of antibiotics

d

The nurse is teaching the patient and family that peptic ulcers are a. caused by a stressful lifestyle and other acid producing factors such as H. pylori b. inherited within families and reinforced by bacterial spread of staphylococcus aureus in childhood c. promoted by factors that tend to cause oversection of acid, such as excess dietary fats, smoking, and H. pylori d. promoted by a combination of factors that may result in erosion of the gastric mucosa, including certain drugs and alcohol

d

The nursing management of the patient with cholecystitis associated with cholelithiasis is based on the knowledge that a. shock-wave therapy should be tried initially b. once gallstones are removed, they tend not to recur c. the disorder can be successfully treated with oral bile salts that dissolve gallstones d. laparoscopic cholecystectomy is the treatment of choice in most patients who are symptomatic

d

The obesity classification that is most often associated with cardiovascular health problem is a. primary obesity b. secondary obesity c. gynoid fat distribution d. android fat distribution

d

This bariatric surgical procedure involves creating a gastric pouch that is reversible and no malabsorption occurs. What surgical procedure is that? a. vertical gastric banding b. biliopancreatic diversion c. Roux-en-Y gastric bypass d. adjustable gastric banding

d

To prevent the infection and transmission of STIs, the nurse's teaching plan would include an explanation of a. The appropriate use of oral contraceptives b. sexual positions that can be used to avoid infections c. the necessity of annual pap tests for patients with HPV d. sexual practices that are considered high-risk behavior

d

What is important for the nurse to include in the postoperative care of the patient following tympanoplasty? a. check the gag reflex b. encourage independance c. avoid changing the cotton padding d. instruct patient to refrain from forceful nose blowing

d

What should a patient be taught after a hemorrhoidectomy? a. Take mineral oil before bedtime. b. Eat a low-fiber diet to rest the colon. c. Administer oil-retention enema to empty the colon. d. Use prescribed pain medication before a bowel movement

d

You are caring for a young woman who has painful fibrocystic breast changes. Management of this patient would include a. scheduling a biopsy to rule out malignant changes b. teaching that symptoms will probably subside if she stops using oral contraceptives c. preparing her for surgical removal of the lumps, since they will become larger and more painful d. explaining that restrictions of coffee and chocolate and supplements of vitamin E may relieve some discomfort

d

Postoperative nursing care for the woman with a gynecologic fistula includes (select all that apply) a. ambulation b. bladder training c. warm sitz baths d. perineal hygiene e. use of stool softeners

d, e

The patient has been diagnosed with benign paroxysmal positional vertigo. The nurse knows that which anatomic area of the ear contributes to this disturbance? 1 2 3 4

three semicircular canals


Kaugnay na mga set ng pag-aaral

What is the longest word you know

View Set

Physiology of Health and Aging: Exam 1

View Set

CompTIA A+ Practice Exam 1 (220-1102)

View Set

Chapter 39: PrepU - Management of patients with oral and esophageal diorders

View Set